Jump to content

Wikipedia:Reference desk/Science: Difference between revisions

From Wikipedia, the free encyclopedia
Content deleted Content added
→‎"Acute in age": one more...
Line 531: Line 531:
Not a request for medical advice, more of an inquiry of whether there are skin cells (that are alive) that need oxygen from the exterior. [[Special:Contributions/72.229.155.79|72.229.155.79]] ([[User talk:72.229.155.79|talk]]) 18:43, 5 January 2013 (UTC)
Not a request for medical advice, more of an inquiry of whether there are skin cells (that are alive) that need oxygen from the exterior. [[Special:Contributions/72.229.155.79|72.229.155.79]] ([[User talk:72.229.155.79|talk]]) 18:43, 5 January 2013 (UTC)
:No, they don't need oxygen, or you couldn't wear a bandage for very long. The problem is that some chemicals will irritate or block pores if left on too long. You can see if the product has a tollfree number to call for advice. [[User:Medeis|μηδείς]] ([[User talk:Medeis|talk]]) 18:53, 5 January 2013 (UTC)
:No, they don't need oxygen, or you couldn't wear a bandage for very long. The problem is that some chemicals will irritate or block pores if left on too long. You can see if the product has a tollfree number to call for advice. [[User:Medeis|μηδείς]] ([[User talk:Medeis|talk]]) 18:53, 5 January 2013 (UTC)
:: What are the consequences of blocked pores? To me it seems it would just stop sweat. [[Special:Contributions/72.229.155.79|72.229.155.79]] ([[User talk:72.229.155.79|talk]]) 19:19, 5 January 2013 (UTC)

Revision as of 19:19, 5 January 2013

Welcome to the science section
of the Wikipedia reference desk.
Select a section:
Want a faster answer?

Main page: Help searching Wikipedia

   

How can I get my question answered?

  • Select the section of the desk that best fits the general topic of your question (see the navigation column to the right).
  • Post your question to only one section, providing a short header that gives the topic of your question.
  • Type '~~~~' (that is, four tilde characters) at the end – this signs and dates your contribution so we know who wrote what and when.
  • Don't post personal contact information – it will be removed. Any answers will be provided here.
  • Please be as specific as possible, and include all relevant context – the usefulness of answers may depend on the context.
  • Note:
    • We don't answer (and may remove) questions that require medical diagnosis or legal advice.
    • We don't answer requests for opinions, predictions or debate.
    • We don't do your homework for you, though we'll help you past the stuck point.
    • We don't conduct original research or provide a free source of ideas, but we'll help you find information you need.



How do I answer a question?

Main page: Wikipedia:Reference desk/Guidelines

  • The best answers address the question directly, and back up facts with wikilinks and links to sources. Do not edit others' comments and do not give any medical or legal advice.
See also:


January 1

Industrial Revolution and Coal

The Industrial Revolution used and began our use of coal. But won't coal be used up? What is the answer of the people who originally began, invented, and developed the Industrial Revolution?

Republicanism (talk) 00:37, 1 January 2013 (UTC)[reply]

Of course sooner or later it will be used up -- but it will not happen for at least 400 years, so this is not an urgent issue at this time. 24.23.196.85 (talk) 00:49, 1 January 2013 (UTC)[reply]
It's unlikely that we could actually consume the remaining coal before we make the planet virtually unlivable...hence we won't run out of coal. If we have sense, we'll stop using it long before it actually runs out. The people who started the industrial revolution were (at the outset) using so little coal, that the issue of if/when we'd ever run out was simply not a problem. It's only as that use accelerated through the next century that the possibility of running out might be considered. SteveBaker (talk) 00:56, 1 January 2013 (UTC)[reply]
[citation needed] "Virtually unlivable" in what way? If you mean smog or acid rain, this can easily be mitigated by scrubbing the sulfur out of the smoke, as is universally done today. 24.23.196.85 (talk) 01:42, 1 January 2013 (UTC)[reply]
I imagine he meant global climate change due to increased atmospheric carbon dioxide. I certainly hope we will have better energy options than coal, before that happens. StuRat (talk) 03:41, 1 January 2013 (UTC)[reply]
[citation needed] The notion that CO2 is responsible for climate change is disputed -- there are many other factors at work here, including an increase in solar radiation, and/or the action of trace gases (especially glyoxal and its derivatives). Also, climate change will not make the planet "virtually unlivable" -- in fact, it can actually benefit the civilized countries (especially Europe and Canada) by increasing agricultural productivity. 24.23.196.85 (talk) 04:05, 1 January 2013 (UTC)[reply]
Sweet racism bro. Those uncivilized brutes in Africa totally deserve what climate change will bring. 24.255.30.187 (talk) 06:33, 1 January 2013 (UTC)[reply]
So you want all of us Americans to cripple our own energy sector and set our economy and culture back by maybe two centuries just because of the off chance that some Stone-age troglodytes might get their land flooded if we don't? What kind of crazy, self-hating thinking is this? 24.23.196.85 (talk) 00:50, 2 January 2013 (UTC)[reply]
At this point, the only ones who deny that CO2 causes climate change are right-wing politicians and any "scientists" on their payroll. StuRat (talk) 06:38, 1 January 2013 (UTC)[reply]
Even among those people, very few deny that CO2 causes warming, since the physical fact that CO2 is a greenhouse gas is indisputable. They just deny that warming has in fact happened on Earth, or if it has, that it's anthropogenic. --99.227.0.168 (talk) 16:58, 1 January 2013 (UTC)[reply]
It is worth noting that with carbon sequestration we might be able to use up all the coal, oil shale, and assorted such goodies without getting overheated - if we are clever enough to do it but not clever enough to outsmart ourselves, that is. (I am primarily thinking of the carbonate formation under "chemical processes" in that article) Wnt (talk) 13:23, 1 January 2013 (UTC)[reply]
Being one of "those people" I think it is clear that the extra CO2 humans do contribute to the atmosphere does contribute to increased warming, but that other factors obviously hugely drown out that signal (look at the warmth of the late middle ages and the Roman era), and that a massive socialist redistribution scheme is not the answer to any problem if an anthropogenic one is ever conclusively demonstrated. I am also hugely in favor of nuclear power, a consumption (i.e., carbon and other materials) tax, and encouraging the development of fusion via tax and patent incentives. That actually seems like a rather sober and objective viewpoint if you ask me. μηδείς (talk) 21:29, 1 January 2013 (UTC)[reply]
Except for the part about fusion power, it is. Fusion is at least fifty years away from being commercially viable; nuclear fission, on the other hand, is not only viable but is actually cost-competitive with coal and natural gas, and is also very safe for the most part (especially with the new generation of super-safe nuclear reactor designs in development right now, like the lead-bismuth-cooled fast-breeder, or the General Atomics graphite-helium reactor). 24.23.196.85 (talk) 01:01, 2 January 2013 (UTC)[reply]
I don't have an educated opinion as to how far distant viable fusion power is (and 50 years is soon) but I am in favor of granting a fifty year patent on it and waving all tax on profits associated with it for that period. That should incentivize it like you can't imagine. μηδείς (talk) 02:46, 2 January 2013 (UTC)[reply]
(ec) And I'm a socialist, so we could squabble all day. Instead, since this is the science reference desk, I'll provide a scientific reference, because there's an overwhelming consensus among climate scientists against your unsupported assertion that "other factors obviously hugely drown out that signal". From the Medieval Warm Period article: "globally the Medieval Warm Period was cooler than recent global temperatures" (with a citation). From the graph in that article, the temperature peak representing the MWP is 0.1-0.2 C in height and a few hundred years in width, whereas the global temperature has risen 0.8C since the beginning of the 20th century. I could not find any temperature estimates of the Roman Warm Period. --99.227.0.168 (talk) 01:05, 2 January 2013 (UTC)[reply]
Your "overwhelming consensus" is that of politically motivated liars who've been caught out over and over. There's no suitable long term sample, just recent flawed cheerleading models for the refuted hockeystick. But do cite away til you feel warm and cozy. μηδείς (talk) 02:42, 2 January 2013 (UTC)[reply]
Even if this is true, this does not prove that CO2 is responsible for the warming trend -- there's a lot of other crap being released into the atmosphere that didn't used to be back then, like methane, nitrogen oxides, all sorts of halogenated hydrocarbons, and the above-mentioned glyoxal derivatives, ALL of which are considered to be much more potent "greenhouse gases" than CO2. Add to that the concurrent increase in solar radiation (which is right now the only plausible explanation for the warming trend currently observed on Mars), and then anyone can see that CO2 is by far NOT the only factor at play here, nor is it even necessarily the biggest. Correlation does not imply causation! 24.23.196.85 (talk) 01:23, 2 January 2013 (UTC)[reply]
If you are asking historically — e.g. what did the people who started the Industrial Revolution think — it's worth pointing out that 1. nobody really consciously started the Industrial Revolution, and 2. the amount of coal is quite vast, even today, and was considered essentially limitless at the time of the Industrial Revolution. So I doubt they worried much about it running out in the short term. --Mr.98 (talk) 03:29, 2 January 2013 (UTC)[reply]
Agreed; there's still a huge reserve of coal under the UK (where the industrial revolution began),[1] it's just a whole lot cheaper to import it from open cast mines abroad. Alansplodge (talk) 18:37, 2 January 2013 (UTC)[reply]

Eliptical Machines

I am using an eliptical machine as a cross training activity for long distance running. I find something about them strange. Why do I not feel sore at all the day after a strenous workout on an eliptical machine (eg doing fast paced intervals or going for a long period of time), but feel sore the next day after a equally strenous run or weight lifting workout? I know they are supposed to be low-impact, but I would expect to feel some muscle soreness. Same thing with swimming. -anon

I'm guessing you're in good shape, so the exercise doesn't overtax your muscles and cause soreness that way. Thus, the only soreness you normally feel is from microscopic injuries caused by impacts. You might want to increase your resistance/weight/reps/time to make it more of a challenge. StuRat (talk) 03:35, 1 January 2013 (UTC)[reply]
Stu's on the right track. It probably also means that you have a good elliptical, or at least one that suits you well. I've used at least three different styles of elliptical and had different experiences with each regarding muscle tiredness, joint sensitivity, and so on. Matt Deres (talk) 04:40, 1 January 2013 (UTC)[reply]
My experiences and opinions. I think elliptical machines are very low impact. Your feet are on the paddles ALL THE TIME. And your muscles are working nearly ALL THE TIME. The machine is quite friendly to your legs. You can you your arms if you really want to. I can usually "run" maybe 1.5 times of distance on a elliptical machine than actually jogging.
This may not be a fair comparison because we have air conditioning in a gym. This is not fair. It can be very hot outdoors. I also drink less when I am jogging. You always have water in a gym.
I think treadmills are friendlier than jogging and elliptical machines are even more friendlier than treadmills. However, jogging outdoors can be much more fun. I just hate to jog in summer. -- Toytoy (talk) 16:05, 1 January 2013 (UTC)[reply]

Did any of you use an eliptical at any point? What is a suggestion as to how much I should increase the duration of my workouts every sesson? Is increasing the duration of my workouts by 2 minutes each sesson reasonable? — Preceding unsigned comment added by 99.146.124.35 (talk) 23:14, 1 January 2013 (UTC)[reply]

We can't give that sort of advice for practical and legal reasons, but you may have a personal trainer or other professional available at your gym who can--do ask them. μηδείς (talk) 02:37, 2 January 2013 (UTC)[reply]
The advice I was given was to slowly increase the workout until I do get sore, or at least feel muscle fatigue. This is basically the "no pain no gain" advice, although I think "pain" is a bit strong. You should feel it, but not be in agony. You might also want to strap weights on your legs or otherwise increase the difficulty level. StuRat (talk) 03:21, 2 January 2013 (UTC)[reply]
Retaining a personal trainer for even a few sessions can really help. I don't think exercise advice is quite at the level of medical advice (which we're not allowed to give), but it's simply common sense: a good personal trainer can watch what you're doing and give you instant feedback about what you're doing right and wrong - and tips for the future. Matt Deres (talk) 23:25, 2 January 2013 (UTC)[reply]

Alexander graham bell

complaint over priority referred to article talk page
The following discussion has been closed. Please do not modify it.

Alexander graham bell did not invent the telephone, that went to an inventor a few years earlier. Putting a sentence saying he invented a working telephone, insults all pioneers and their inventions. You should change the reference to include the Alexander Graham was created the first telephone the concept and design for which was invented by.... I cant recall the person who actually invented the phone. — Preceding unsigned comment added by 124.148.227.176 (talk) 03:35, 1 January 2013 (UTC)[reply]

This belongs on that talk page. However, Leonardo da Vinci had diagrams of helicopters [2], but his designs wouldn't fly, so should he be credited with their invention ? StuRat (talk) 03:38, 1 January 2013 (UTC)[reply]
Bell's article says he is credited with inventing the first practical telephone, a big difference from what you're complaining about. Invention of the telephone lists various candidates for the telephone's daddy. Clarityfiend (talk) 05:29, 1 January 2013 (UTC)[reply]

Closing--OP has been refererred to article talk page. μηδείς (talk) 16:35, 1 January 2013 (UTC)[reply]

Lizzie Velazquez - A Rare Condition

Speaking in a strictly academic capacity with perfect propriety, my first question is what actually is the name of the rare condition Lizzie Velazques has? I can't seem to find anything on wikipedia. I don't think she has a page. All the articles, everything I see on google/youtube, no one actually mentions the name of the condition. They just keep saying that it is such a rare condition that only 2 or 3 people in the world are known to have it. Any useful informative references on the disease will be useful. Second question, from what I understand, she weighs just 60 pounds and has no adipose tissue, and cannot create muscle, store energy, or gain weight. She has no body fat or muscle tissue but then how come her mammary glands are so swollen? Maybe it is my ignorance of human physiology but I thought that subcutaneous adipose tissue (according to wikipedia anyway) gives breasts its shape and size. So if she's not supposed to have any fat or muscle in her body, then how come her breasts are so big and shapely? At least to me they look unusual compared with the rest of her stature. Is it known if she ever got artificial enhancements/supplements or if the condition takes away fat/muscle from certain parts of the body and leaves the rest alone? Thanks! - Looking for Wisdom and Insight! (talk) 10:49, 1 January 2013 (UTC)[reply]

Second Google hit references [3] which says neonatal progeroid syndrome is the current best guess (we only have progeria written so far); during her childhood de Barsy syndrome had been suspected, but she didn't have learning difficulties. It says she is currently being studied by Professor Abhimanyu Garg, MD, at the University of Texas Southwestern Medical Center in Dallas, who came up with the NPS diagnosis. Wnt (talk) 13:17, 1 January 2013 (UTC)[reply]
Perhaps they are just falsies or implants, allowing her to look normal in at least one respect. StuRat (talk) 03:17, 2 January 2013 (UTC)[reply]

Air refueling stealth fighters

F-22s and F-35s are capable of air refueling.

Why do they need air refueling? Certainly, they need extra fuel to extend their ranges.

But unlike many other airplanes, these airplanes do not have hardpoints, as a result, they can always add a central hardpoint for an external fuel tank.

I know that external fuel tanks are radar reflectors.

But air refueling also gives away a stealth fighter's position. The tanker airplane is a much larger radar reflector and air refueling is slow.

On the other hand, external fuel tanks are lighter, more efficient and they can be dropped.

Why don't stealth fighters use external tanks? -- Toytoy (talk) 15:54, 1 January 2013 (UTC)[reply]

From Wikipedia's article on the F-22: "the wings include four hardpoints, each rated to handle 5,000 lb (2,300 kg). Each hardpoint has a pylon that can carry a detachable 600 gallon fuel tank or a launcher holding two air-air missiles."
Our article on the F-35 don't mention it, but this link seems to point ot at least two of the hardpoints being plumbed for droptanks. Since the F35 is still years away from being in service, take any information regarding it with a grain of salt though.
WegianWarrior (talk) 16:18, 1 January 2013 (UTC)[reply]
You can carry more weapons without a heavy external fuel tank. And carrying an external fuel tank for the home trip into a combat zone makes you slower and less maneuverable. Air refueling is usually done in an area considered relatively safe from enemy attack. (Have planes ever been attacked during the procedure?) See more in Aerial refueling. PrimeHunter (talk) 16:45, 1 January 2013 (UTC)[reply]
And using a safe area for refuelling (which is necessary mostly because the tankers are vulnerable) means that if the enemy does realise the fighter's presence, they'll likely lose track of it once it detaches from the tanker and moves into the hot zone. With the exception of a tiny handful of convert assassination and first-strike operations, the purpose of stealth isn't to conceal the presence of the aircraft, but to prevent enemy missiles from successfully engaging it. The air defence system around Baghdad in 1991 was perfectly well aware that there were F117s around (once stuff started exploding) but they couldn't get their radar-guided weapons to acquire a lock on them. I guess there may be some interest in a stealthy refuelling system, to support the tiny number of cloak-and-dagger strikes, and perhaps to maintain stealthy drones. -- Finlay McWalterTalk 17:09, 1 January 2013 (UTC)[reply]
Is there any reason why there couldn't be a stealth refueling plane, or why refueling itself couldn't be stealthy? I mean, stealth doesn't require a plane to perform like a fighter, does it? I imagine they don't tell us everything they waste money on... Wnt (talk) 19:13, 1 January 2013 (UTC)[reply]
It seems that it could in principle be done, but there's not a lot of good reason to do it. (Note that even very 'stealthy' aircraft would be vulnerable during midair refuelling; the aircraft performing such a manoeuvre are necessarily flying relatively slowly, carefully maintaining constant speed and altitude. I also don't know what wrinkles might occur in developing a stealthy fuelling boom.) Consider the biggest 'stealth' aircraft ever built, the Northrop Grumman B-2 Spirit 'stealth bomber'. Normally, it carries up to 76 metric tons of fuel, and up to 23 tons of ordnance; it has an operational range of a whopping 11,100 km. Conceptually, there's no serious difficulty with stripping out the bomb racks and replacing them with another 23 tons of internal fuel storage, deliverable up to a third of the way around the world. I suspect that if you cut into the B-2's own existing tanks to trade range for deliverable capacity, you'd probably get close to the same fuelling capabilities as a KC-135.
For reference, the McDonnell Douglas KC-10 Extender carries a maximum fuel load of 160 tons and has an operational range of 7,000 km; its still-in-service predecessor the Boeing KC-135 Stratotanker carries up to 91 tons and has a range of 2,400 km. (The actual amount available for refuelling other aircraft depends on how far the tanker has to fly and how long it has to loiter; our articles don't break out the specific amount of fuel available for transfer from these tankers versus the amount required for their own use.) Our hypothetical B-2 Stratotanker costs about $800 million (plus development costs for the midair fuelling variant), whereas a KC-135 costs about $40 million. TenOfAllTrades(talk) 21:35, 1 January 2013 (UTC)[reply]
Our article on the F22 says: "Range: >1,600 nmi (1,840 mi, 2,960 km) with 2 external fuel tanks" - so it does indeed have the capability to extend it's range in that way. Furthermore: "Fuel Capacity: 18,000 lb (8,200 kg) internally, or 26,000 lb (11,900 kg) with two external fuel tanks"...and that's the problem. With in-flight refuelling, you can almost double your range - with external tanks you gain at most maybe 40% more range. (Also, those tanks cause drag and extra weight - which means that the aircraft is less fuel-efficient when carrying them - so they clearly don't extend range by as much as you'd expect from the raw numbers). Furthermore, you can (and routinely would) refuel on the way out to the target and on the way home - which triples your range. SteveBaker (talk) 02:08, 2 January 2013 (UTC)[reply]
I noticed that F-22's long flights over the Pacific Ocean require external tanks. I still think that carrying multiple external tanks are more reasonable than air refueling.
You just drop the tanks and then you're OK. -- Toytoy (talk) 03:40, 2 January 2013 (UTC)[reply]
About vulnerable stratotankers, it might be less of a vulnerability than you think, with all those fighters moving back and forth. It could be an issue if there was another (non-US) power using stealth aircraft, trying a sneak attack on the refueling fighters and the KC herself, but as of now, non-stealth fighters are detected from 100's of miles away, and refueling doesn't last that long. They could intercept any non-stealth intruder without penalties to their weight/drag/stealth.
Keep in mind that applying a layer of stealth skin to a drop tank is not an option. You don't want to give away stealth technology, lest the enemy copy or defeat them.
About stealth refuelers, some jets including the Tornado GR.4 have been converted to tankers in small numbers, so it is likely that some stealth planes have been converted, too. - ¡Ouch! (hurt me / more pain) 08:33, 2 January 2013 (UTC)[reply]
Ahem, Panavia Tornado. - ¡Ouch! (hurt me / more pain) 08:57, 2 January 2013 (UTC)[reply]
Of course we're missing another use of inflight refuelling - which has little to do with extending range. When you want to carry the maximum possible amount of munitions. The maximum weight that the aircraft can take off with is considerably less than the amount it can fly straight and level with. So one trick is to take off with a minimal fuel load (thereby allowing more payload to be carried) and to almost immediately refuel when at altitude. Once at altitude, the total of fuel plus payload can safely exceed the maximum take-off weight. SteveBaker (talk) 15:27, 2 January 2013 (UTC)[reply]
And there may be a day when you need to attack a target that's more than 920 miles away. The RAF's 1982 Operation Black Buck raids involved a round trip of 8,000 nautical miles, each bomber requiring 11 tanker aircraft. In the Gulf War, "on 16 January 1991, a flight of B-52Gs flew from Barksdale AFB, Louisiana, refueled in the air en route, struck targets in Iraq, and returned home – a journey of 35 hours and 14,000 miles round trip". Alansplodge (talk) 18:32, 2 January 2013 (UTC)[reply]

Gravitational and magnetic force

why we can't relate gravitational force with magnetic force? since most of the living and non living things contains elements that can be attracted by magnetic force. 116.202.68.193 (talk) 16:31, 1 January 2013 (UTC)[reply]

I would attempt an answer to that question if only it made any sense... Dauto (talk) 17:58, 1 January 2013 (UTC)[reply]
I'm guessing they mean "replace it" as in artificial gravity in a space station.--Gilderien Chat|List of good deeds 18:39, 1 January 2013 (UTC)[reply]
They are similar in some ways, but not similar enough. For starters, everything accelerates exactly the same under gravity (this has been tested to one part in ten trillion), but some magnets are much stronger than others. Plus, magnetism can't explain the bending of starlight or the anomalous precession of Mercury's orbit. -- BenRG (talk) 18:40, 1 January 2013 (UTC)[reply]
Ben, I'm not sure I understand what you mean by "everything accelerates exactly the same under gravity" - are you referring to Galileo's experiment showing that different masses accelerate at the same rate (and thus fall at the same rate) due to gravity? Because if so the obvious rebuff is, just as some magnets are much stronger than others, some masses are much more, er, massive than others... (so the distance between two 1-kg masses finitely far apart in a vacuum with no other forces decreases faster than that between a 1-kg mass and a 1-g mass in the same situation, for example).
@OP: I'm surprised no one's mentioned this, but there is the fact that magnets have poles, and we have never discovered a magnetic monopole (this not to say they don't exist, of course), and precisely the opposite, gravity is only ever attractive so far as we can observe, there is no "negative mass" that repels the mass we observe (even antimatter attracts matter via gravity, likewise darkmatter). Even if there were, it wouldn't line up with how we observe like charges and magnets to repel, because it would have to be opposites that repel and likes that attract (to see why this is, consider the sun, the earth, and the moon. They all attract each other, so they are likes, because if they were opposites this couldn't be the case) 72.128.82.131 (talk) 04:20, 2 January 2013 (UTC)[reply]
General relativity and electromagnetism do fit together rather nicely, in the form of Kaluza–Klein theory. There's also gravitoelectromagnetism, although that's just an analogy that only works in an approximate sense under certain conditions. Red Act (talk) 22:54, 1 January 2013 (UTC)[reply]

Any studies on whether whale strandings may have analogue in human stampedes?

67.243.3.6 (talk) 17:09, 1 January 2013 (UTC)[reply]

  • The dynamics are unsimilar, whales are not crowded onto the beach or crushed or trampled by other whales. (My personal bet is that there is a microorganism to blame which spends part of its life cycle on the beach, similar to other parasites which drive their hosts to odd behaviors like rabies or fungi that cause snails and ants to climb to high locations in order to be eaten or to spread spores in the wind.) μηδείς (talk) 21:18, 1 January 2013 (UTC)[reply]
"Citation needed". :) Though it is a creative idea, and could be true. I would suspect that there could be a common basis - stranding of whales and dolphins are widely though not unanimously blamed on loud sonar arrays (see whale beaching), and I suspect that loud noise contributes greatly to the confusion in some human stampedes. But I can't find a reference for that, either... Wnt (talk) 02:03, 2 January 2013 (UTC)[reply]
If you are asking me regarding my personal guess, I already quoted myself. Do you want me to put it on my talk page so someone else can quote me here? μηδείς (talk) 02:29, 2 January 2013 (UTC)[reply]
Meideis' ridiculous speculation aside, see beached whale#Causes for more credible explanations. Beaching of single whales is much more common than beachings of multiple whales, and is usually caused by injury or disease. Multiple-whale beachings are sometimes caused by strong social cohesion--one whale sends out a distress call while close to the shore, and other whales follow. By contrast, human stampedes are caused by panic or disasters, and death is usually by asphyxiation. If you've been to any major event, like Toronto's New Year countdown, you know how easy it is for the crowd to squish you when lots of people are pushing. I don't see any obvious connection between the two, but if the OP wants to point out what kind of similarities he's thinking of, he might receive a better answer. --99.227.0.168 (talk) 05:41, 2 January 2013 (UTC)[reply]
Hiding behind an IP does not give you any special privilege to ridicule the posts of others here. ←Baseball Bugs What's up, Doc? carrots06:26, 2 January 2013 (UTC)[reply]
This is a reference desk, not a "make up your own theories and post them" desk. I see no rule that says IP users cannot correct non-IP users when the non-IP user posts no references and the IP user does. --99.227.0.168 (talk) 07:07, 2 January 2013 (UTC)[reply]
Your term "ridiculous speculation" does not belong here. ←Baseball Bugs What's up, Doc? carrots07:15, 2 January 2013 (UTC)[reply]
Well, editors really ought to be providing referenced facts rather than "personal bets". However, "It is by politeness, etiquette and charity that society is saved from falling into a heap of savagery". Alansplodge (talk) 17:53, 2 January 2013 (UTC)[reply]

Anyhow, back to the point. I've had a fairly good trawl through Google and found Keeping Together in Time, or the Natural History of Social Cohesion. A Critical Evaluation of W.H. McNeill's New Book (1995) is an article that seeks to compare a new work on human social cohesion with mammalian behaviour. The Google search result for the page says "It (presumably group behaviour) may take precedence over all other forms of behaviour, as for instance in lemming migrations or mass stranding» of whales; but much more often it is integrated with other modes of social interaction." You can't view the article without subscription. Sorry, that was all I could find. Alansplodge (talk) 18:17, 2 January 2013 (UTC)[reply]

Thanks. It should be noted that lemmings behavior has been greatly exaggerated. 67.243.3.6 (talk) 14:17, 3 January 2013 (UTC)[reply]

Understanding the EPR paradox

In the article on the EPR paradox an explanation is given on how the experiment was or can be performed. The part about this experiment I'm trying to get is not the faster-than-light aspect, but rather the certainty or not of a state.

I'm very familiar with 3D vector math, so the part that puzzles me is when they talk about the spin that can be seen as "a superposition of two states". I view the spin as an "axis" (about which the electron "spins"). Surely this must lead to an infinite number of possible states, corresponding to directions. Is the article saying that the experiment can be set up in such a way that only those two states are possible?

And then, when the measurements along a particular axis is made, are we talking about projections (like dot product) or is there some quantum effect that causes the vectors to become 100% parallel to the measured axes? Or is some quantum effect simply making all measurements binary (with no possibility of zero or small values) along all possible axes? 105.236.57.198 (talk) 19:03, 1 January 2013 (UTC) Eon[reply]

This is a basic fact of spin (quantum mechanics). Spin can be along any axis, but when measured, it always has one of a few discrete values according to which way you looked. As I understand, remeasuring it from another angle has a chance of getting the other value, depending how wide the angle - and if you remeasure again from the first angle, it could then be different from what it was the first time! So defining the axis is ... difficult. Most of the physics types will say don't even think about it in classical terms, it's just a number, but we find this ... unsatisfying. Not when it still sorta kinda is like something you can picture. Note that the general idea that angular momentum is quantized is absolutely fundamental to quantum mechanics - people might talk about how, say, an electron has quantized "energy", but really, it has equally spaced increments of angular momentum that determine its energy because if it had any more or less with a given angular momentum it wouldn't be in orbit. Every photon carries the exact same angular momentum, the reduced Planck constant, and that is what an electron changes by when it flips from one spin state to the other, just as when it goes up a level in energy. Wnt (talk) 19:16, 1 January 2013 (UTC)[reply]
Thanks for an excellent summary. Actually, I couldn't figure it out from that description because it doesn't go all the way. I found this article by a user here that continues the explanation to the point that I fully understand the weirdness of entanglement. It doesn't appear to be an official article, but I think it would be great if that was included. The official article merely "tells" one how it works. That explanation convinces one by tracing the path to the conclusion. 105.236.57.198 (talk) 20:10, 1 January 2013 (UTC) Eon[reply]
I used this question as an excuse to revamp something I wrote ages ago. It's now at User:BenRG/Bell's theorem as a game show. Let me know if it's helpful. -- BenRG (talk) 06:53, 3 January 2013 (UTC)[reply]
By the way, the 99%/98%/96% argument in the actual article is wrong. I'll mention it on the talk page.It's okay. There are classical counterexamples to the claim "if A and B measured on two particles are 99% correlated, and likewise B and C, then A and C are at least 98% correlated". But there are no counterexamples if you also require that A and A, B and B, and C and C are 100% correlated. -- BenRG (talk) 06:54, 3 January 2013 (UTC)[reply]

Astronomical naming conventions

I get a bit confused over the names of some things. Our star, and its planets, are collectively known as the "solar system", but "solar" comes from Sol, the name of our star. What, then is the general name of such a system called? Star system apparently refers to systems of multiple stars, while planetary system seems to exclucde the star (the article specifically mentions The Sun and its planetary system).

Similarly, what would a "solar flare" on any other star be called? A stellar flare? 90.193.232.232 (talk) 19:24, 1 January 2013 (UTC)[reply]

Basic commutativity. If the sun is a star, then a star is a sun, right? Poetically I've heard of the "children of other suns". I would say that there are many solar systems but only one Solar system. The same astronomers probably have a pet "Dog" :) But I'm not an astronomer, sorry. Wnt (talk) 20:01, 1 January 2013 (UTC)[reply]
This seems to me just like first having the Moon, and then finding out that there are other natural planetary satellites and calling them moons. There are lots of moons but only one Moon. I've often heard of other stars referred to as other suns, although there is only one Sun. So the analogy would be that there are many solar systems but only one Solar System. Words' meanings evolve as semantic needs evolve. Duoduoduo (talk) 22:25, 1 January 2013 (UTC)[reply]
We have adequate terminology now, if only I we were able to force convince people to use it. In the long term, better usage will evolve. Then people will move to other stars and the provincials will start talking about earthquakes on their new planet and sunstorms on their new star. Depressing. μηδείς (talk) 02:27, 2 January 2013 (UTC)[reply]
An earthquake is when the earth shakes, not when the Earth shakes, so "earthquake" is fine on Mars, when the good red Martian earth starts quaking beneath your pressure-suited feet. --Trovatore (talk) 05:39, 2 January 2013 (UTC)[reply]
As noted below, Moonquakes and Marsquakes and even Sunquakes are attested. μηδείς (talk) 17:18, 2 January 2013 (UTC)[reply]
The terms earth, moon, planet and sun are all carryovers from the assumption that we are the center of the universe... which, for most practical purposes, we are. The earth is the ground we stand on, the "moon" is cognate with the "month", the sun is cognate with words that mean "shine". Medeis is right that if and when we migrate to other planets and stars, terms will evolve. If you're on Mars, there's a reasonable chance you'd call it a "Marsquake", but if we establish a civilization there, in some future generation they might come up with different names, as the notion of it being the realm of the god of war would seem silly. ←Baseball Bugs What's up, Doc? carrots06:23, 2 January 2013 (UTC)[reply]
No, earth meaning dirt is the older usage. The planet is named after the dirt. There's nothing wrong with talking about an earthquake on Mars, as it is indeed a quaking of the earth that's there. (Not sure Mars actually has tectonics at all, but that's a side issue.) --Trovatore (talk) 07:48, 2 January 2013 (UTC)[reply]
Sure, and that's where the planet's name came from. At that time, no one knew there was soil on other planets (not for sure, anyway). You may find Quake (natural phenomenon) of some interest. ←Baseball Bugs What's up, Doc? carrots07:56, 2 January 2013 (UTC)[reply]
You're missing the point, and at this point I have to suspect you're being deliberately dense. It's not called an Earthquake, meaning the planet shakes. It's called an earthquake, meaning the dirt shakes. Mars has earth too. --Trovatore (talk) 08:20, 2 January 2013 (UTC)[reply]
Well, who should I believe, you or the wikipedia article? ←Baseball Bugs What's up, Doc? carrots08:23, 2 January 2013 (UTC)[reply]
The article does seem to source "moonquakes" and such, but surely those words were chosen a little facetiously. If not, it's just dumb. --Trovatore (talk) 08:25, 2 January 2013 (UTC)[reply]
I've certainly heard the term "moonquake" before. I can't vouch for the others as such. Maybe you would call it a Marsquake if you were observing from earth, and an earthquake if you were actually standing on Mars, or maybe just a "quake". Mars certainly has solid ground and some kind of soil or ground (let's not say "dirt"), but there's no guarantee that citizens of Mars would use the term "earth" to indicate the surface, at least not right away. ←Baseball Bugs What's up, Doc? carrots08:41, 2 January 2013 (UTC)[reply]
It's interesting that the Latin terra is likewise used for both the land and for the planet.[4] The two concepts are closely linked. ←Baseball Bugs What's up, Doc? carrots08:43, 2 January 2013 (UTC)[reply]
'Marsquake', How Earthquakes On Mars Could Sustain Alien Life is a Huffpost Tech article dated Feb 24, 2012 that is based on an interview with a NASA-affiliated scientist. Others based on the same report and with the same approximate date are [5] and [6]. Duoduoduo (talk) 13:42, 2 January 2013 (UTC)[reply]
However, the abstract and title of the study that those news articles are reporting on refer to "paleomarsquakes" and "marsquakes". Duoduoduo (talk) 13:58, 2 January 2013 (UTC)[reply]

Geology of Mars. Duoduoduo (talk) 12:54, 2 January 2013 (UTC)[reply]

If we ever got to live on a planet orbiting another star, I'm pretty sure we'd still say things like "What a pretty sunset!" and "Oh, look! A full moon. How romantic!" - and not get into tangled new vocabulary like "Alpha-Centauri-rise" or "A full Triton". It seems that when uncapitalized, "sun" and "moon" refer to whichever star and moon you happen to be nearest to and when capitalized "Sun" and "Moon" refer to the Earth's sun and moon. It's only in the last hundred years that it's mattered - and even then, not to many people and not in most conversations. But capitalization would rapidly get confusing in speech - so "Sol" and "Luna" are probably words we might take up if we lived elsewhere than on Earth and we needed to talk specifically about the Earth's sun and moon. In places like NASA, "earth" (as in dirt) is carefully called "regolith" in cases where ambiguity might result - and the Mars missions talk about "sols" instead of "days" to avoid confusion. Our language adapts as need arises. Words like "earthquake" are corner cases - but using terms like "moonquake" and "marsquake" makes it difficult to write sentences like "The comparative study of earthquakes on different celestial bodies is a branch of seismology." - you can't say "The study of quakes..." because the word "quake" can mean ground shaking other than of a geological origin (see Wiktionary). SteveBaker (talk) 15:14, 2 January 2013 (UTC)[reply]

Rydberg molecular hydrogen

A few more questions about Rydberg atoms forming Rydberg matter. (I also list some assumptions in case they're wrong...) Feel free to answer any part independently.

1. I assume it is possible for two Rydberg atoms of hydrogen to interact in such a way that they are not immediately disrupted, to form a Rydberg molecule of hydrogen with two protons and two electrons. ([8] describes one with three hydrogens, which I shall not even try to figure out.)

2. My recollection is that Rydberg atoms of hydrogen have something like n=20, maybe 50, and the electron can - somehow - be visualized as an isolated blob of negative charge revolving around a distant nucleus in a Keplerian orbit.

3. Suppose a Rydberg atom has a perfectly circular orbit, and does not emit a photon while being observed. Or suppose it has a perfectly linear orbit, with the electron falling straight through the photon on every oscillation. Can you call one a "50s orbital"? The other a "50p orbital"?

4. Suppose two Rydberg atoms are collided so that two electrons are somehow orbiting two protons in a stable configuration. What would that look like?

5. Is the bond qualitatively different if you have one electron with "n=49" and another with "n=50", as opposed to two with 50? Can you visualize the Pauli exclusion principle somehow because the spins have to be opposite for them to fall in the same orbital? Or is there a way to write a molecular orbital formula for the two electrons so that they sail neatly one after the other along the same course?

6. Is the angular momentum of each electron quantized in relation to each of the two protons? In relation to the other electron? In relation to the whole system? Do these conflicting demands for simultaneous quantization force them out of a perfectly elliptical orbit as measured?

Wnt (talk) 19:57, 1 January 2013 (UTC)[reply]

A Rydberg atom is just an ion and an extra electron that's bound to it (i.e. has negative total energy) but isn't as close as the other electrons. You can certainly form bound states of two ions, and you can always in principle introduce another particle whose kinetic energy is small enough that it's bound to the ions, and I suppose that would be a Rydberg molecule, if you want to call it that.
The electron orbitals that you see in textbooks are not the only possible orbitals. They're simply a basis for the possible orbit(al)s, i.e. for the possible wave functions of bound electrons. If an electron is bound to a nucleus then (in a suitable approximation) you can always express its motion as a sum of basis orbitals, even if the electron is localized and following a quasiclassical orbit. Quasiclassical orbits don't have a single letter or number; they're some complicated sum (superposition) of infinitely many orbitals with many different values of n and ℓ. But you don't have to use that basis; you can just think of them as quasiclassical orbits, which is mathematically equivalent and probably more sensible in this case. In general these systems are going to look (for a brief time) sort of like a classical gravitational system.
The quasiclassical electron will be spread out in orbital-angular-momentum space in accordance with the uncertainty principle. But the orbital angular momentum space is discrete/quantized (since it's compact) so it's actually spread out over a bunch of discrete momenta. If you measured its orbital angular momentum (which is not really possible but is theoretically possible) you would force it into an orbit of fixed angular momentum, which would be spread all around the atom in position space and hence nonclassical.
Incidentally "nonclassical" here really refers to classical particle theory. A lot of what I said above still holds in classical wave theory. You can still do the decomposition into basis orbitals, there's still an uncertainty relation, the angular frequency is still quantized, etc. -- BenRG (talk) 05:52, 3 January 2013 (UTC)[reply]
Thanks for setting me straight. I suppose that if you could make the measurement, and thereby disperse an electron into a symmetric shell, this would be the most iconic illustration of the Heisenberg uncertainty principle that I can think of. Wnt (talk) 15:06, 4 January 2013 (UTC)[reply]

Brain question

I was wondering, since the brain is mostly controlled via electrical impulses, and you can mess with peoples heads by shocking parts of the brain (I think?), what would happen if you were to open up a persons skull and pour something like saline solution, or some sort of liquid that conducted electricity onto it? Would the brain short circuit? What if it were just dribbled instead of coating the whole of the surface? Gunrun (talk) 23:36, 1 January 2013 (UTC)[reply]

Neurons, including those in the brain, do indeed work on electric pulses, but they are not like currents flowing through a conductor: the ions are "pumped" chemically not electrically. So concepts like "circuit" in the electrical sense (and a fortiori "short circuit") simply do not apply. --ColinFine (talk) 00:14, 2 January 2013 (UTC)[reply]
However, it is possible to "mess with peoples heads by shocking parts of the brain" - see Electroconvulsive therapy. Don't try this at home. Alansplodge (talk) 00:56, 2 January 2013 (UTC)[reply]
A less-dramatic approach is transcranial magnetic stimulation. --Mr.98 (talk) 03:25, 2 January 2013 (UTC)[reply]
The brain is already immersed in saline solution, and it works just fine (in most people, anyway) -- so just on the face of it, the OP's suggestion won't work. 24.23.196.85 (talk) 01:07, 2 January 2013 (UTC)[reply]
Presumably the balance of the various ions that the brain uses would be kinda critical - so while dumping extra saline in there wouldn't short anything out - it would screw up all of the concentrations - and that might well be fatal. SteveBaker (talk) 01:45, 2 January 2013 (UTC)[reply]
The answer to this is complex. The conductance of the extracellular medium could vary, and would affect neural signalling. action potentials cross the membrane; even so, the voltage, and also specific ions that have to move, affect the opening of more channels up and down the axons and dendrites. Much of this happens inside the neuron where the change in medium might not affect it (depending on what exactly you did to it). See [9] for some basic biophysics of this - but I won't claim to have gone through it all. Wnt (talk) 01:51, 2 January 2013 (UTC)[reply]
Given the sophistication of the question, I think the prior answers (no, nerves and the brain don't work that way) are sufficient. The OP should read brain, neuron, and action potential, and then come back and ask for help if he wants it at that point. μηδείς (talk) 02:24, 2 January 2013 (UTC)[reply]
Replacing the CSF with saline would kill you, as Steve said. It doesn't have the calcium that is absolutely required for neuronal activity (and indeed for survival of all other cells. Cells are pretty good at concentration calcium, but stores would be depleted over time as they migrate across the concentration gradient. Fgf10 (talk) 11:45, 2 January 2013 (UTC)[reply]

Thanks for the answers and links and stuff guys, time to do some reading! Gunrun (talk) 23:04, 2 January 2013 (UTC)[reply]

CO2 laser - output power versus input.

I have a CO2 laser (100 Watt) which is rated for 23mA of power at the power supply. The power supply was shipped with the current limiter set somewhere down around 8mA...so the laser wasn't performing very well. So now I've adjusted the limiter to provide the 23mA required - and I'm definitely getting more "oomph" out of the laser now - although the only measure I have of that is how well it cuts holes in things - and that's not linear with the laser energy anyway.

The question is: Is the amount of output laser power directly proportional to the input - or is there something more complicated going on?

Another issue is that these lasers degrade slowly over time (I believe the seals are imperfect so you get impurities leaching into the CO2 gas - which reduces the power you get out of the machine). Would it be reasonable to assume that the current consumed by the laser would be a reasonable measure of that deterioration - or does it consume just as much input current but produce less laser energy?

If it helps, this is a water-cooled device.

TIA SteveBaker (talk) 23:52, 1 January 2013 (UTC)[reply]

To the first question, no. The maximum power output is at the point where internal resistance of the power supply is equal to load. --Gilderien Chat|List of good deeds 02:16, 2 January 2013 (UTC)[reply]
That would be the maximum power output of the power supply - that's not what I'm asking. The power output of the laser is my concern. Not all of the electrical energy going into the thing emerges as useful light - much is wasted in heating the thing up (hence the need to water cool it). I'm interested in the relationship between the current going into the tube and the light energy coming out of it. SteveBaker (talk) 14:53, 2 January 2013 (UTC)[reply]
It's impossible to answer this question without details of the laser construction and its cooling. Why don't you perform an experiment to see how much it heats up identical volumes of infrared-opaque fluid (or just water) for the same duration at different power input levels? 70.59.14.20 (talk) 18:26, 2 January 2013 (UTC)[reply]
It's not exactly a trivial experiment for someone who doesn't have access to a fully equipped lab! It's also tough to do - glass is opaque to the IR light coming from the laser and is etched by it - and you can't risk stray reflections from things in the path of the laser because they tend to set light to things on the other side of the room! I rather hoped someone might know the general form of the relation between input power and light output. SteveBaker (talk) 22:09, 2 January 2013 (UTC)[reply]
Fire the laser downward into water in a metal dewar flask for a second, then put a thermometer in it? 70.59.14.20 (talk) 01:21, 4 January 2013 (UTC)[reply]
For an ideal laser, there is a "threshold" input power, below which the laser won't lase. Once the input power is above threshold, the laser output should ideally rise linearly with the input power. In practice, the output rises linearly with input for a while and then rolls over due to heating of the gas and other effects that reduce efficiency.
Degradation in performance over time may be due to erosion of the electrodes rather than contamination of the gas. This might cause the voltage required to get a given current to increase. Changes in the electric field distribution due to the wear on the electrodes would probably cause the laser to produce less power at a given current over time.
Make sure you wear proper laser safety goggles while operating the laser. Even a tiny fraction of a 100 W beam can blind you.--Srleffler (talk) 17:49, 4 January 2013 (UTC)[reply]
Ah! That's what I needed. So it's linear with the input power once it's lasing and then up to some point where overheating takes over (which should be well-controlled with the chilled water supply)? That's exactly what I needed to know! Thanks!
(And, yes - when the case of the machine is open, we wear special polycarbonate safety goggles that are specifically designed to filter out light at the frequency of the laser. When the machine is running (it's a laser cutter BTW), the case is light-tight except for a dark-tinted polycarbonate lid - which, again, is specifically designed to filter out the harmful light. Even the reflection of a reflection of the beam is enough to cause eye damage! This thing can happily slice through a half inch of solid wood! SteveBaker (talk) 17:54, 4 January 2013 (UTC)[reply]


January 2

New comet and meteor shower

C/2012 S1#Associated Meteor Shower says that the Earth will pass through the path of this new comet, and may experience a meteor shower. The comet is on a hyperbolic orbit. It seems to me that any particles that come off the comet will be on a similar orbit and would not be around to form a meteor shower. What is right? Bubba73 You talkin' to me? 02:55, 2 January 2013 (UTC)[reply]

Diagram of a comet showing the dust trail, the dust tail (or antitail) and the gas tail. NASA
Yes the dust tail will be on a similar orbit, but it is expected to be very long, and part of it will have arrived on our orbit when we come around to it.--Shantavira|feed me 09:11, 2 January 2013 (UTC)[reply]
The article says "Its orbit is nearly parabolic", which presumably means that it is elliptical with an eccentricity of just below one. Bubba, why do you say it is hyperbolic?--did you read that somewhere else? "Near parabolic" appears in two different refs sourced by the article: New Scientist and Astronomy Now. Also, I don't think that the nature of the orbit (hyperbolic or elliptical near parabolic) should even affect the likelihood of a meteor shower -- the comet is from the Oort Cloud, which is way out there, so it will be a long time before the comet itself reappears even if elliptical. So as Shantavira says, any meteor shower would come from the dust tail of the current passage. Duoduoduo (talk) 13:10, 2 January 2013 (UTC)[reply]
The article gives the eccentricity as 1.0000015, which makes it hyperbolic. If particles come off the comet, wouldn't they continue on their orbit? That is, how will it make an annual shower? Bubba73 You talkin' to me? 14:55, 2 January 2013 (UTC)[reply]
Our article Comet tail has a diagram which I have copied here, showing a dust trail following the comet's orbital path, and a dust tail or antitail coming outwards away from the sun, but at a different angle to the gas tail. I'm no expert, so I stand to be corrected. Alansplodge (talk) 17:44, 2 January 2013 (UTC)[reply]
So it does. But I can't find anywhere that says the meteor shower would be annual. Where do you see that? Duoduoduo (talk) 17:44, 2 January 2013 (UTC)[reply]
Pass. Sorry, I'm already out of my depth here! Alansplodge (talk) 17:46, 2 January 2013 (UTC)[reply]
I took out the part about the possible meteor shower this morning because of wp:ball. Look back a little in the article history. It does say that the Earth will pass through the orbit annually and it did say that the first meteor shower could be in January 2014. That seemed to imply an annual meteor shower. Bubba73 You talkin' to me? 17:54, 2 January 2013 (UTC)[reply]
Unfortunately what you left in in that section was pointless without the point that you deleted. I've put it back in with a correction to conform with the source -- the source doesn't say anything about "annually". It's okay to give a statement of what astronomers predict -- many of our astronomy articles do that. Duoduoduo (talk) 18:10, 2 January 2013 (UTC)[reply]
One amateur astronomer predicted that. The Jan 2014 date will be a year after the comet passed by. I asked NASA's meteor expert on his facebook page, but I don't know if he will get back to me. Bubba73 You talkin' to me? 18:41, 2 January 2013 (UTC)[reply]
Scratch that about a year after - I forgot that it is 2013 already. Bubba73 You talkin' to me? 18:45, 2 January 2013 (UTC)[reply]
(unindenting) About the "hyperbolicity" of the orbit in question, it isn't very obvious. If I got my math right, the angle between the two asymptotes in radians is about the square root of 2(ex - 1) if the excentricity ex is close to 1. That is about .0017 radians or .01 degrees. I.e. it would not show up as hyperbolic to the naked eye even if its faintness were a non-issue.
Oops, 0.1 degrees. Stupid math fail. - ¡Ouch! (hurt me / more pain) 10:49, 4 January 2013 (UTC)[reply]
Bringing up the Oberth effect in reverse, i.e. inelastic collision, the solar wind could be enough to slow dust to a velocity below solar escape velocity, i.e. make its orbit elliptic without affecting the solid comet in any measurable way. Funny how that reduces to the same mass/surface ratio issue as the "animals in free-fall" topic below. - ¡Ouch! (hurt me / more pain) 06:47, 4 January 2013 (UTC)[reply]
One minor correction a point made above: ISON will not be a periodic comet; in fact, it is currently on an ejection orbit, which will put it out of the Solar System within a few decades. Humans won't be laying eyes on this comet between 2014 and whenever we figure out the Alcubierre drive. I did a lot of reading on this topic last week when I thought I had reason to believe that it was a reappearance of Newton's comet, which I later found out a lot of others had suspected as well. JPL numbers put that theory out of commission, though. Evanh2008 (talk|contribs) 11:55, 4 January 2013 (UTC)[reply]
Alcubierre drive is FTL, thus overkill. With a hyperbolic but near-parabolic trajectory ("orbit" has a wrong sound to it IMO - it won't "be back"), it'll slow down to 1 km/s or less eventually. v is proportional to r-1/2 in a circular orbit and 1.414... times that on a parabolic escape course, that is v = (1650/r)1/2 if r is in AU and v is in km/s (right? I hope my math is right this time...). One would need quite high delta v, but even a Pioneer could catch up to it, given the right planetary alignment. It's not about the speed but about finding that damn snowball in the big black void. - ¡Ouch! (hurt me / more pain) 18:15, 5 January 2013 (UTC)[reply]

Plants on Mars

To be clear, according to [10] no known plant can grow on Mars without artificial assistance. And yet I wonder. The atmosphere of Mars has about 600 Pa of pressure of carbon dioxide; by comparison, unless I went wrong somewhere, 0.039% of Earth's 101300 Pa is 39.5 Pa. So Mars has plenty of atmosphere for plants (provided they don't try to do respiration...). The climate of Mars includes temperatures up to as much as 81 degrees Fahrenheit. Debatably, conditions in Hellas Planitia can even allow the formation of liquid water (though we have a citation needed there) and the crater includes glaciers. And we know at least microbes can survive beneath glaciers. [11] But potentially, we might be even more ambitious: boreal trees survive freezing even in liquid nitrogen, if given a chance to prepare. [12] It is considered at least possible to develop UV-resistant plants in response to destruction of Earth's ozone layer.. [13] So ... could the NASA site be too pessimistic? Do any of the (would-be) exobiologists think that plant life, perhaps (with a little tinkering) even plants we recognize, could actually be able to grow on Mars? Wnt (talk) 03:31, 2 January 2013 (UTC)[reply]

Evidence of liquid water has been observed on Mars, but we don't really know how long it lasted. If any plants were brought to Mars and exposed to the naked atmosphere, wouldn't any water in them be prone to evaporate rather quickly? Evanh2008 (talk|contribs) 06:46, 2 January 2013 (UTC)[reply]
The Martian atmosphere has an average pressure of about 6 millibars, as I recall. By my math that would put the boiling point of water right around negative five C (I may have gotten some numbers jumbled there, though). I guess if you could find a place on Mars where there was moisture and temperatures consistently below that mark, plants should grow just fine. Evanh2008 (talk|contribs) 06:49, 2 January 2013 (UTC)[reply]
Yes, to retain water they might look more like a cactus, and perhaps use oils instead of water for their circulatory system. I certainly wouldn't say it's impossible for some form of life to exist on Mars, just that it would have to be fairly different from Earth life. However, some form of microbe living in underground damp soil seems more likely than plants, to me. StuRat (talk) 06:55, 2 January 2013 (UTC)[reply]
To clarify, I'm speaking of plants from Earth here (presumably deliberately introduced) to keep things a little less open-ended. And yes, retaining moisture seems like it would be a huge problem, though if rooted in a glacier I'd think they might have a chance of preserving water balance. But I don't actually know if any plants can endure very low pressure conditions like this; I found a cite that vegetables can survive brief vacuum. [14] Wnt (talk) 07:08, 2 January 2013 (UTC)[reply]
Boiling point can not be lower than melting point. Martian conditions are close to the triple point of water, but, taking into account the low partial pressure of water vapor in the martian atmosphere, any melted water will likely start boiling almost immediately. Ruslik_Zero 07:27, 2 January 2013 (UTC)[reply]
Lower than local melting point, no. Lower than 'sea level earth melting point, of course it can. The triple point on Mars has to be different than the triple point on earth. That's how science works. Evanh2008 (talk|contribs) 07:47, 2 January 2013 (UTC)[reply]
First, the melting point of water lowers with rising pressure. Second, the triple point is a characteristic of water only, not ambient conditions. The phase diagram of water is same everywhere in the Universe. Ruslik_Zero 08:05, 2 January 2013 (UTC)[reply]
You seem to be confusing the concept of the triple point with the temperature at which thermodynamic equilibrium between the phases occurs. On Earth the temperature needs to be about 273 K. I'm not about to compute what it is on Mars, but it has little to do with the fact that water under a six-millibar (.1771 inches Hg) atmosphere will boil at -5 C, give or take a bit. Everyone is welcome to break out a calculator and check for themselves. Evanh2008 (talk|contribs) 08:21, 2 January 2013 (UTC)[reply]
Okay, I see what you mean now; my numbers were off. In theory the boiling point is around -41 C, but you're right about the triple point. I guess there is no true boiling on Mars then? Just sublimation? Evanh2008 (talk|contribs) 08:41, 2 January 2013 (UTC)[reply]
You're forgetting plants still need oxygen as well. Yes, I know (most) plants produce an excess of oxygen, but when not actively photosynthesising (ie in the dark) they need oxygen for respiration just as much as humans do. So just having the right partial pressure of carbon dioxide isn't enough. I guess you could just keep them in constant light, but I don't know what effects that would have. Fgf10 (talk) 11:41, 2 January 2013 (UTC)[reply]
Well, I was assuming that whenever they were in the dark they'd need to be frozen/inactive anyway. In theory I suppose they could store redox potential in some other form than oxygen. Wnt (talk) 15:32, 2 January 2013 (UTC)[reply]
Hydrogen peroxide? Plasmic Physics (talk) 01:36, 4 January 2013 (UTC)[reply]
I was thinking more of some sort of melanin, if a battery-like potential could be charged from local minerals, or oxidation of hematite to goethite, etc. But if the plant can tank them effectively, I suppose peroxides would work. Wnt (talk) 03:27, 4 January 2013 (UTC)[reply]

At what distance from our sun would Sirius and the sun have the same brightness?

Pluto? The Oort Cloud? How many AUs? Thanks.Rich (talk) 04:51, 2 January 2013 (UTC)[reply]

The absolute magnitude of the Sun is 4.83, of Sirius—1.42. They will have the same brightness if the ration of distances to them from an observer is 10(4.83-1.42)/5=4.81. The distance to Sirius is 2.6 pc. So this translates to the distance from the Sun of about 2.6/5.81=0.45 pc or about 90,000 AU. Ruslik_Zero 07:57, 2 January 2013 (UTC)[reply]
Which translates to beyond or near the presumed outer reaches of the outer Oort cloud (depending on how you define its extent). — Quondum 08:44, 2 January 2013 (UTC)[reply]
That calculation elided that the brightness goes down as the square of distance and the absolute brightness is got by multiplying by 0.4. Half of 0.4 is 0.2 which is 1/5. Dmcq (talk) 13:09, 2 January 2013 (UTC)[reply]
Pardon my confusion, but why did you get 4.81 and then divide 2.6 by 5.81? Why doesn't it end up 2.6/4.81, which makes 0.541 pc? Nyttend (talk) 20:52, 2 January 2013 (UTC)[reply]
You seem to be correct, there is apparenty a slip in the original calculation, making the distance more like 110,000 AU. — Quondum
The absolute magnitude of the Sun is +4.83, the absolute magnitude of Sirius is +1.42, i.e. 3.41 magnitudes lower. A source 5 magnitudes lower than another is 100 times brighter, so Sirius is 100(4.83-1.42)/5=23.1206 times brighter than the Sun (at the same distance). The apparent brightness varies with distance according to the inverse-square law, so Sirius would need to be sqrt(23.1206) = 4.81 times as far away as the Sun to appear equally bright as the Sun - i.e. at 4.81AU (end of calculation). For comparison, Jupiter is 5.2AU from the Sun. --catslash (talk) 02:25, 3 January 2013 (UTC)[reply]
4.81 is ratio of distances. So, the total distance should be divided by 4.81+1=5.81. Ruslik_Zero 08:28, 3 January 2013 (UTC)[reply]
I think the OP was assuming that the Sun and Sirius are fixed - where would the observer be for them to be equally bright? Bubba73 You talkin' to me? 02:38, 3 January 2013 (UTC)[reply]
Stupid of me. Then for the two distances from the observer to be in the ratio 1:4.81, her distance from the Sun is (1/(1+4.81))2.64pc = 0.454pc = 93700AU --catslash (talk) 10:36, 3 January 2013 (UTC)[reply]
No, you had the ratio right. Bubba73 You talkin' to me? 23:30, 3 January 2013 (UTC)[reply]
Or, a more general question: given two objects of absolute magnitudes X and Y, at what distance from X are the apparent brightnesses equal? --jpgordon::==( o ) 07:52, 3 January 2013 (UTC)[reply]
If the distances of X from the observer is x and the distance of Y is y and the total distance is d, and the absolute magnitudes are Mx and My then you need to solve simultaneously
The latter simplifies to
and substituting and solving for x
(note that Mx and My are the other way around from in the preceding equation), which gives 93750AU from the Sun for the original problem. --catslash (talk) 10:36, 3 January 2013 (UTC)[reply]
The distance ratio defines an ellipsoid with one focus at the Sun and the other behind it as seen from Sirius. Most of the responses seem to concern the intersection of that ellipsoid with a sphere with Sirius at its center and Sol on its surface. —Tamfang (talk) 19:11, 30 June 2013 (UTC)[reply]

Physics and chemistry questions

I have a number of questions which are not being answered neither by my seniors nor my teachers. However, some of them are easy .It is better to ask some of them at one time instead of one by one.

  1. Why do like charges repel each other and unlike charges attract each other ?
  2. Why does water boil at a lower temperature at high altitudes ?
  3. Why does cooking anything in water take longer time on mountain top than at sea level ?
  4. When we twist our fingers, it produces sound. Why is it so ? Is there any suitable word instead of "twist" ?
  5. Chemical formula of copper sulfate pentahydrate is "CuSO4·5H2O". What does ·5H2O represent ? Is it correct to say · as dot ?

Sorry, I have asked too much. BTW Thank you and Happy New Year. Want to be Einstein (talk) 07:31, 2 January 2013 (UTC)[reply]

1) I'm not sure that we really know why, it's just a fundamental observation that they do. It might be explained in terms of quantum mechanics, string theory, and such, but ultimately you still come to a point where we say "it's that way just because it is".
3) Since water boils at a lower temperature, that keeps the water at that lower temperature, assuming the steam is free to boil off (so not in a pressure cooker, say). It takes longer to cook things at a lower temperature, and, if the temperature is too low, it may never cook.
4) Do you mean cracking knuckles ? I believe that's caused by cavitation.
5) That means it has 5 water molecules loosely bound to the central molecule. Others can add details and how you say it. StuRat (talk) 07:47, 2 January 2013 (UTC)[reply]
Regarding item 2, Boiling point explains that the boiling point varies depending on the atmospheric pressure. The lower the pressure, the lower the boiling point. ←Baseball Bugs What's up, Doc? carrots08:22, 2 January 2013 (UTC)[reply]
The "like repels like" thing with charges isn't something we have an answer for - all we know is that this is what happens. There are many situations in science where we have complete knowledge of how things behave without any understanding of why. About the best we can come up with is the Weak Anthropic Principle - which basically says that if the universe worked the other way around, then (presumably) atoms would not have formed and therefore there would be no stars, planets, people, Wikipedias and Wikipedia Science Reference Desks. Hence, only those universes where physics work the way our universe does will have people around to ask the question. It's an unsatisfying answer - but it's really all we have.
Water boils at the temperature at which the vapor pressure of steam equals or exceeds that of the air above the surface. At high altitudes, the air pressure is lower, so the vapor pressure of the steam can be lower - which happens at lower temperatures. Since cooking is mostly about getting the food up to some desired temperature, most things cook more slowly at high altitudes. Contrast this with a pressure cooker which deliberately increases the pressure inside the pot to allow the water inside to reach higher temperatures without boiling so that foods cook more quickly than they otherwise would. SteveBaker (talk) 14:38, 2 January 2013 (UTC)[reply]
I would quibble with StuRat's answer to #5, in that while water of crystallization includes water which is loosely bound (coordinated) to a central atom, this may be written (if the structure is known) in a different notation, e.g. Rhodium (III) chloride = RhCl3(H2O)n. Sometimes waters not directly bound to the central atom nonetheless turn up in a structure, as with the [Cu(H2O)4]SO4·H2O mentioned in the article; but all are included when written generically in the dot notation the questioner used. There's a bit of inconsistency in the article regarding Glauber's salt - our article says only 8 of the 10 waters are directly bound, but the water of crystallization article writes it as if all 10 are. Anyway, my point is that if you look at the crystal structure the waters aren't always clearly associated with one particular molecule; it can be more like an aqueous solution that has simply become concentrated enough to take on a crystalline solidity. Wnt (talk) 15:29, 2 January 2013 (UTC)[reply]

I read this in "Microsoft Student with Encarta Premium 2007"
In one example of the electromagnetic force, two electrons repel each other because they both have negative electric charges. One electron releases a photon, and the other electron absorbs it. Even though photons have no mass, their energy gives them momentum, a property that enables them to affect other particles. The momentum of the photon pushes the two electrons apart, just as the momentum of a basketball tossed between two ice skaters will push the skaters apart.
This explains why like charges repel each other. I searched whole encyclopedia but it does not provide any information about unlike charges. Is this explanation correct ? Answer to Q.4 given by StuRat mean same as what I thought. One more thing I want to know is that Is it harmful to crack knuckles ? Want to be Einstein (talk) 16:07, 2 January 2013 (UTC)[reply]

Cracking knuckles is not harmful as far as we know yet, according to Cracking knuckles#Effects, except that "habitual knuckle-crackers were more likely to have hand swelling and lowered grip strength", which doesn't address which is cause and which is effect. From the article: "it remains unclear if knuckle cracking is in itself associated with any impaired hand function." Duoduoduo (talk) 17:52, 2 January 2013 (UTC)[reply]
With regard to the electromagnetic force question, the photon is the force carrier, a virtual particle exchanged in an exceedingly short period of time, which causes repulsion between like charges. The phenomenon is the same where attraction is concerned, in that photons are exchanged - it's just not as easy to describe intuitively as the basketball analogy does for repulsion. It is important to remember that the virtual particle model is just an interpretation of what happens, and although it gives correct results under enough conditions to be useful, it is only one possible way of looking at it, and has its limitations. 72.128.82.131 (talk) 18:27, 2 January 2013 (UTC)[reply]
The Encarta explanation is a really weak one. A logically-minded reader would have to assume that two positrons would also exchange a photon to repel each other - and then immediately ask why the photon emitted by a positron attracts rather than repelling a regular electron? A photon is a photon...there isn't a matching anti-particle - so how does the electron "know" what emitted the photon in order to know whether to be attracted or repelled. Perhaps there is some good science here - but there is no sign of it in that explanation! SteveBaker (talk) 22:02, 2 January 2013 (UTC)[reply]
Probably because the very very short explanation does not explain what a virtual photon is, or how to intuitively understand it! The electron doesn't emit a single, real photon. It emits a continuous wavelike set of virtual photons, most of which cancel out and have no effect on the universe. When encountering another charged particle, also swimming in another sea of virtual photons, some of those virtual particles cancel out, and others constructively interfere; and real momentum and energy can be exchanged, depending on how they all cancel out. Now, why a sea of imaginary semi-non-existing particles is any more satisfying than a field theory, I do not understand; but modern physicists generally agree that the model of virtual particles that cancel out is a better model. This model resolves more questions, and leaves fewer holes in the explanation, than the classical field theory we learn in first-year physics. More on this: electrostatic Coulomb force, explained as exchange of virtual particles. Rest assured, our article only barely explains this topic; physics educators generally recognize that to learn this topic, about four or six years of advanced university-level mathematics and physics are prerequisite. Nimur (talk) 22:10, 2 January 2013 (UTC)[reply]
Question 1 is fairly tautological. "Like" charges repel because we call charges that repel each other "like". If they attracted each other, we'd call them "unlike". - Nunh-huh 01:32, 3 January 2013 (UTC)[reply]
I disagree. If you group objects into two groups, based on their behavior, there are two possible attraction scenarios:
1) Everything in group A attracts everything in group B, while everything in group A repels everything else in group A and everything in group B repels everything else in group B. This is "like repels like".
2) The reverse. Everything in group A repels everything in group B, while everything in group A attracts everything else in group A and everything in group B attracts everything else in group B. This is "like attracts like".
Rearranging the groups can't change scenario 1 to scenario 2 or vice-versa.StuRat (talk) 02:51, 3 January 2013 (UTC)[reply]
Fourteen other possibilities:
  • A repels all, B repels A and attracts B
  • A repels all, B attracts A and repels B
  • A repels all, B attracts all
  • A repels A and attracts B, B repels all
  • A repels A and attracts B, B repels A and attracts B
  • A repels A and attracts B, B attracts A and repels B
  • A repels A and attracts B, B attracts all
  • A attracts A and repels B, B repels all
  • A attracts A and repels B, B repels A and attracts B
  • A attracts A and repels B, B attracts A and repels B
  • A attracts A and repels B, B attracts all
  • A attracts all, B repels all
  • A attracts all, B repels A and attracts B
  • A attracts all, B attracts A and repels B
But if A vs B is a continuum (e.g., quantity of charge or mass) rather than a dichotomy, I guess we're stuck with Stu's two cases. Is there an example of Stu's second case in particles? —Tamfang (talk) 19:49, 30 June 2013 (UTC)[reply]
  • 1) Has been answered by several people above, but the "why" question doesn't have an answer. Some "fundemental properties" come in two opposing varieties which repel each other. That's a description of how they work, the why boils down to either the anthropic principle or "because that's the way God made it" or something. There's not a good answer for that kind of "why" question, it just is.
  • 2) Has also been answered. But to say it in simplest terms: Something is a liquid or a solid because the forces holding it together exceed the forces trying to tear it apart. Thermal energy is the primary motivation for molecules to fly apart (molecules which are separated from each other is called a "gas". The process of a liquid turning into a gas at maximum effect is called "boiling"). There are several things holding molecules together in liquid form: intermolecular forces are one, but external pressure also plays a part. If you lower the external pressure on a liquid, you release some of the force holding it together, so you make it easier for thermal energy to tear the molecules from each other, and thus lower the boiling point. At high elevations, you have less air above you, so it weighs less, so it exerts less pressure.
  • 3) As noted, at higher elevations, water boils at less than 100 degrees C. That means that the hottest the water can get is lower than the hottest the water can get at lower elevations. Lower maximum temperature means longer cooking times.
  • 4) Joint cracking is often caused when ligaments or tendons rub against each other or against the bones they attach to. It can also be caused by the synovial bursa shifting or rearranging rapidly (the aforementioned cavitation). Some people mistakenly thinks its because the bursa are breaking or rupturing; that is incorrect: if you ruptured your bursa, you'd be in a serious amount of hurt.
  • 5) This is called either Water of crystallization or water of hydration. The specific nature of the way in which the water is bound to the molecule in question is ambiguous and inconsistent between various nomenclatures; there isn't a consistent way of noting it, despite what some claim above, in the difference between •xH2O and (H2O)x. It means that individual water molecules are stoichiometrically bound up in the formula unit in some way; that is water acts like an atom or ion or other moiety in the formula of the substance. The water molecule can be bonded in a coordinate bond to, say, the metal as a ligand or it can occupy a specific locale in the crystal lattice as part of the unit cell. The formula is not always clear on how the water is bound, (in either notation), but it is important because the water needs to be dealt with stoichiometrically when, say, weighing the substance out on a scale and calculating how many moles of the substance you have.
Does all that help? --Jayron32 03:15, 3 January 2013 (UTC)[reply]

It's a theorem of relativistic quantum field theory that like charges of even-spin forces attract and like charges of odd-spin forces repel. Gravity and the Higgs force are the only fundamental even-spin fields (spin 2 and spin 0 respectively). The other fundamental forces, including electromagnetism, are spin-1. No one knows why only those forces with those spins exist (except for gravity, which seems necessary). Still, it's interesting that there can't be a field similar to the electromagnetic field in which like charges attract. -- BenRG (talk) 05:10, 3 January 2013 (UTC)[reply]
Another fundamental factor in the universe is the speed of light. Is there an explanation for why the speed of light is what it is and not some other speed? That is, it's about 186,000 miles per second. Why is it that, rather than 150,000 or 200,000 miles per second, for example? ←Baseball Bugs What's up, Doc? carrots11:18, 3 January 2013 (UTC)[reply]
It's basically a measure of how fast we think. Traditional units are chosen for human convenience. A brain is about a decimeter across and it takes about a second to "have a thought", so the time scale of thought is about 3 billion times the light travel time across the brain. (The neuronal signals travel slower than that, but the speed of light sets the standard for all other motion.) It's a random-looking number (299,792,458 m/s) because there's some arbitrariness in the selection of convenient units. -- BenRG (talk) 20:55, 3 January 2013 (UTC)[reply]
I don't follow. The speed of light is a constant (in a given medium). What is it about the nature of light that defines what its speed is? ←Baseball Bugs What's up, Doc? carrots00:44, 4 January 2013 (UTC)[reply]

Thanks to all, every question was answered more than I expected. Want to be Einstein (talk) 17:26, 3 January 2013 (UTC)[reply]

You're welcome. May we mark this Q resolved ? StuRat (talk) 18:42, 3 January 2013 (UTC)[reply]

Of course. Want to be Einstein (talk) 19:35, 3 January 2013 (UTC)[reply]

Resolved

Professional engineers

Are professional engineers used in any aspect of film production directly or indirectly? In which areas? 176.27.208.210 (talk) 11:20, 2 January 2013 (UTC)[reply]

Certifying the building drawings of the buildings (which are used in filming, as they house sets) on the studio lots. 67.163.109.173 (talk) 11:39, 2 January 2013 (UTC)[reply]
There are many different kinds of engineer, but see film crew and particularly sound engineer.--Shantavira|feed me 12:10, 2 January 2013 (UTC)[reply]
Any movie with computer graphics (which these days is almost any movie!) is likely to have a bunch of professional software engineers working for them in various capacities. SteveBaker (talk) 14:16, 2 January 2013 (UTC)[reply]
We have an article, Professional engineer, which explains that in some countries like the United States, there are engineers who are licensed by the state or other government unit, and are allowed to plan projects that could impact public safety, such as electrical wiring in larger buildings, the structure of buildings, water systems, etc. This kind of licensure would not be needed for those aspects of film production that helps to make the film pleasing to the audience. Jc3s5h (talk) 14:31, 2 January 2013 (UTC)[reply]
Ah - yes. If we're talking about licensed professional engineers - then that's a different matter. I doubt that there are many jobs in the film industry that require such a license - although it might be that the strong union rules in the film industry could require a license in some cases. The use of the term "professional engineer" to denote someone in one of those niche engineering jobs that requires a state license is really annoying! I'm an engineer - and I get paid to do it - so I'm definitely a professional. So why can't I call myself a "professional engineer"? Weird! SteveBaker (talk) 14:48, 2 January 2013 (UTC)[reply]
Steve, whether you are a professional or not, and whether you are an engineer or not, you can't "directly or indirectly use or cause to be used as a professional, business, or commercial identification, title, name, representation, claim, asset, or means of advantage or benefit any of, or a variation or abbreviation of, the following terms:" (...) "professional engineer" - per Texas Occupations Code, Title 6, Chapter 1001. Similar laws apply in North Carolina, and in California, where I have also been professionally employed as an engineer, but never as a Professional Engineer. This is sort of an old-fashioned rule that was intended to prevent fraudsters from building shoddy constructions while pretending to be civil engineers; it has very limited applicability to things like electronics and software engineering. Nimur (talk) 18:38, 2 January 2013 (UTC)[reply]
Pretty sure any engineer is a "professional" - I sure hope I never meet any structure or gadget designed by a nonprofessional engineer! 72.128.82.131 (talk) 18:30, 2 January 2013 (UTC)[reply]
See Aggie Bonfire... Wnt (talk) 21:06, 2 January 2013 (UTC)[reply]
The letters I like best are MICE ;-) It stands for member of the institute of civil engineers. One can also become a chartered mathematician or information technologist in Britain. Dmcq (talk) 13:34, 3 January 2013 (UTC)[reply]

90.219.64.4 (talk) 20:48, 6 January 2013 (UTC)== Logic ==[reply]

Isnt everything logical, even creativity? I agree that being logical inhibits creativity as creativity requires thinking beyond things we already understand but the end product of creativity is logical. 176.250.156.24 (talk) 15:38, 2 January 2013 (UTC)[reply]

How does serendipity fit into your argument? — Preceding unsigned comment added by 217.158.236.14 (talk) 15:56, 2 January 2013 (UTC)[reply]
Serendipity is perfectly logical. There's always a probability that an event which is happy for some people will occur. 176.250.156.24 (talk) 18:18, 2 January 2013 (UTC)[reply]
I'm not sure you understand the definition of serendipity. It means "happy accident". Accidents aren't logical. — Preceding unsigned comment added by 217.158.236.14 (talk) 11:16, 3 January 2013 (UTC)[reply]
Why not? There's always a chance accidents will happen. 90.219.64.4 (talk) 14:59, 3 January 2013 (UTC)[reply]
You are equating logic with chance? Saying that random and logic are the same?217.158.236.14 (talk) 17:13, 3 January 2013 (UTC)[reply]
People (and many of their beliefs and actions) are not logical, as Spock liked to remind us.--Shantavira|feed me 16:04, 2 January 2013 (UTC)[reply]
Thats because people aren't perfect. We act based on emotion and our beliefs are formed by past experiences, and possibly genetics. Sounds logical to me. 176.250.156.24 (talk) 18:18, 2 January 2013 (UTC)[reply]
In the real world, logic doesn't always apply, as it requires complete information to draw conclusions. We rarely have complete info when asked to decide, so must use other methods. As Kirk once convinced Spock, "there are times when the logical approach is an illogical one". StuRat (talk) 22:48, 2 January 2013 (UTC)[reply]
In your real world example, isn't that where creativity comes in? As someone said above creativity doesn't contradict logic. 90.219.64.4 (talk) 15:14, 3 January 2013 (UTC)[reply]

Logic means starting from an assumption and following it to what it implies. Some creativity involves coming up with clever assumptions. Some creativity involves finding previously unthought-of ways to go from certain assumptions to interesting conclusions. And some creativity has nothing to do with logic -- e.g. if I come up with a new painting or a new melody.

One needs to distinguish between "logical" in the sense of following the principles of logic to go from assumptions to conclusions, and the looser use of "logical" to mean "not contradicting logic". If I come up with a new melody, I'm not using the principles of logic, and I'm not contradicting them either. Duoduoduo (talk) 18:54, 2 January 2013 (UTC)[reply]

  • There's no inherent conflict between emotion and logic. Without emotion, a Vulcan would never bother to get out of bed in the morning, since he simply wouldn't care. Spock obviously cares, he just doesn't like to show it. That's all a shortcoming of Gene Roddenberry's thought, and has nothing to do with logic, which is simply not acting or arguing contradictorily. μηδείς (talk) 23:03, 2 January 2013 (UTC)[reply]
Whoa there, I agree that there is no inherent conflict between emotions and logic, but I can't see the logical leap to "you need to "care" to get out of bed"... I think that's making too many assumptions, does a computer "care" to wake up when you press the on button? Vespine (talk) 00:56, 3 January 2013 (UTC)[reply]
Which makes more sense to you, to argue that Vulcans are programmed, or to argue about Vulcans? The essence of logic is non-contradiction μηδείς (talk) 03:28, 3 January 2013 (UTC)[reply]
Not sure what you mean. I just didn't buy going from "no conflict between logic and emotion" to "X" would not get out of bed without emotions. Nothing specific about vulcans. Vespine (talk) 04:40, 3 January 2013 (UTC)[reply]
I agree it's not clear he wouldn't get out of bed (if he doesn't care enough to get out of bed, maybe he also doesn't care enough to stay in bed) but I do recall lots of times when Spock would openly express things that I call emotions. He would say, for example, that he preferred some outcome; isn't that an emotion? Interesting was one of his catchwords; interest is also an emotion. --Trovatore (talk) 09:09, 3 January 2013 (UTC)[reply]
Or "fascinating", or the raising of one eyebrow. It's been a long time, but I don't think it was represented that Vulcans are emotionless, but rather that they are in control of their emotions. Spock, of course, was half-human. But Vulcans seem to have a strong sense of morality and ethics, which is hard to explain on logic alone. (And realistically, there have been countless screenwriters for Trek, which tends to muddy the continuity waters.) ←Baseball Bugs What's up, Doc? carrots11:15, 3 January 2013 (UTC)[reply]
I don't think vulcans are completely logical. They use it to overcome certain emotions and conflicts but they themselves aren't very logical. If anything, the Borg are more logical as they have no emotion. 90.219.64.4 (talk) 14:59, 3 January 2013 (UTC)[reply]

But can't the end result of anything be thought of as logical? As mentioned above creativity doesn't contradict logic although it does require going beyond logic. Although actions based on emotions may not seem logical in the wider context, it could be argued that the action is logical for the person feeling the emotion as its the emotion which is causing them to act in such a way. 90.219.64.4 (talk) 15:14, 3 January 2013 (UTC)[reply]

  • Yes, literally, without emotion there is no motivation to move. We get out of bed in the morning because we want to assuage our hunger, or we dislike the notion of crapping ourselves, or we want to get to work on time so we can keep earning money to buy the things we want, all emotions. Even if we say we are acting on force of habit, that is simply the force of past emotions. Of course the emotions involved aren't experienced strongly the way a murderous rage is, and we hardly notice them, given we are adults with long years of practice and some levels of distance and abstraction between the action and the relevant urges. That wasn't always the case. A baby in a crib will scream to get out of it just as it will scream from hunger pains or an ear ache. Consider how you would feel if you had to go to the bathroom and found you were tied to the bed or temporarily unable to move. You may not be aware of it consciously, but in the end there is no intentional action without urges and the desire to satisfy them. Spock is a caricature of a classical stoic, not an emotionless automaton. The latter idea and the belief that logic is the opposite of emotion are mistakes based on a naive misunderstanding of human nature. μηδείς (talk) 17:34, 3 January 2013 (UTC)[reply]
"without emotion there is no motivation to move" - I don't think that's necessarily true. If you have some general, lofty goals (perhaps the idea that preservation of your species into the future is valuable) - then there can still be reason to get up in the morning, get into your big shiney starship and start kicking the butt of evil-doers. In StarTrek, we know that the Vulcans had such goals. Didn't Spock's last words include "The needs of the many outweighs the needs of the few" ? Take a few of that kind of over-arching statement and enter them into the chain of logical thinking as axioms. Wouldn't that result in much the kind of behavior one sees in the Vulcans most of the time.
Mathematically speaking, logic can't do a damned thing without a few axioms to work on. Even logical systems as basic as arithmetic and set theory require axioms. ("There exists a number who's value is zero" and "Every number has a successor" for example). So to know what a totally logical being would do upon awaking each morning depends entirely upon what axioms are inserted into that logical system.
But even in a world where those lofty goals are not present, one could logically say "It makes no logical difference whether I get up or not - so it's a free choice and therefore a random outcome is as good as any other - so I'll flip a coin - and...<flip>...it's heads, so OK, I'll get up!". But if that person takes a longer view: "It makes no difference whether my species survives into the future or not - so I'll flip a coin to decide whether that's something I should strive to achieve"...and from that point onwards, the character has all of the motivations needed to get up in the morning and do great things.
The idea that Vulcans are emotionless is kinda stupid anyway. Emotions are just biochemical signals in the brain - flows of chemicals that are placed there by sensors detecting events in the outside world - evolved over millions of years to help us pass our genetic information onto the next generation. Denying them isn't particularly logical - although recognising and controlling them where higher brain function and logic deems necessary is extremely valuable...and a close approximation of what real-world humans mostly manage to do.
SteveBaker (talk) 16:48, 5 January 2013 (UTC)[reply]
are emotions logical?

nicotine inhaler

I remember back in the 1990s I used to see commercials for a nicotine inhaler that was for quitting smoking and a alternative to patches and gum. This was available over-the-counter. This was a different thing than the E-cig. I don't believe it was electronic. Now all of a sudden it seems to have disappeared from the marketplace in the United States, what happened to it?--Jonharley667 (talk) 17:22, 2 January 2013 (UTC)[reply]

The NICORETTE® Inhalator is still available in the UK, but I haven't seen anybody using one for years. The only US product that I could find (after a very quick search) was the NICOTROL Inhaler which is "Prescription-Only". Alansplodge (talk) 17:32, 2 January 2013 (UTC)[reply]
I am certainly not a medical expert, but a nicotine inhaler sounds totally daft to me. As a general rule, a drug is more addictive if injected ior inhaled, than when eaten, because injecting or inhaling enable high concentrations to reach the brain in seconds. Contrast methodone injected versus swallowed. A nicotine inhaler would continue nictine adiction with full grip on you, whereas a patch would at least have a chance of curing physical addition as the amount of nicotine released slowly decreases over time. Of course the best way to quit is to just make up your mind and stop when you are sick. You can then assign the withdrawal symptoms to whatever made you sick (eg flu). Failing that just quit. Wickwack 120.145.8.219 (talk) 00:58, 3 January 2013 (UTC)[reply]
The point is that nicotine addiction, in itself, isn't harmful. It's the tar which is consumed along with the nicotine that causes cancer. So, if you can decouple the two, and avoid introducing any new chemicals into the lungs, people can continue to get their nicotine fix without dying from cancer or other lung diseases. I'm a bit worried about the combo of hot temperatures and plastic delivery devices, though, as that may create toxic plastic fumes. StuRat (talk) 04:26, 3 January 2013 (UTC)[reply]
Now, hold on. Nicotine#Toxicology would disagree with you on your first point. Nicotine, isn't itself carcinogenic, at least not in the way that the tar and other nasties in tobacco is, but nicotine is not itself a benign substance. Various nicotine replacement therapies (patches, gums, inhalers, etc.) are substitutes for tobacco use in the same way that Methadone is a substitute for heroin: it's less nasty than the thing it is substituting for, but it isn't candy. Nicotine is harmful enough on its own, but it is less harmful than smoking or chewing is. --Jayron32 04:31, 3 January 2013 (UTC)[reply]
Well, you can overdose on it, sure, and pregnant women should avoid it, but if you keep the dosage reasonable, it's about like caffeine, not a bad way to keep awake and focused. In an ideal world, nobody would need either, but this is the real world. StuRat (talk) 06:27, 3 January 2013 (UTC)[reply]
Nicotine#Side Effects also lists potential negative outcomes under non-overdose conditions. --Jayron32 06:40, 3 January 2013 (UTC)[reply]
Careful, Jayron. StuRat normally comes back from a rebuttal with some attempt to prove the rebuttal wrong, citing some dubious website if necessary, or trying to change the subject. If you disagree again, he'll come back again, getting a bit silly. Meanwhile, strangely, the Wiki article you cited doesn't cover nicotine's well known role in triggering TIA's, strokes, heart attacks, and other circulatory issues. It's not thought to be the prime cause but it can be strongly argued that the same applies to the role of smoking in lung cancer. Wickwack 124.178.177.194 (talk) 07:23, 3 January 2013 (UTC)[reply]
Actually, it's you who makes dubious, unsupported claims, like that Australians find 100% humidity comfortable: Wikipedia:Reference_desk/Archives/Science/2012_December_24#Suitable_conditions_for_ventilating_a_residential_property_to_reduce_humidity. StuRat (talk) 18:47, 3 January 2013 (UTC) [reply]
Unsupported claims? Pot calling kettle black there, Stu. What you haven't accepted or understood, and what makes your posts silly (like your claim in http://en.wikipedia.org/wiki/Wikipedia:Reference_desk/Science#Hobbitses "An elephant might die just from falling over" that you continued to argue after given videos of elephants falling over without any such problem), is that a) 100% rel humidity and very close to it occurs in various parts of the World at various times; b) humans have spread all over the World and get used to just about anything that doesn't violate natural limits; c) as pointed out by someone else, not me, in theory so long as the wet bulb temperature is below body temperture, humans can lose heat; I actually claimed that humans need a temperature low enough that the wet bulb is considerably below body temperature in order to be comfortable. Wickwack 60.230.194.70 (talk) 23:31, 3 January 2013 (UTC) [reply]
There is little doubt that nicotine has harmful effects, but my understanding, for which I offer no warranty whatsoever and claim no particular expertise, is that it's much less harmful, everything taken into account, than smoking. If I am right about that, then the focus on the e-cig as a way to quit nicotine strikes me as Puritanical (in the contemporary understanding of that word, which of course has little to do with the historical Puritans). Lots of folks who have no desire to stop using nicotine, per se, are interested in it as a less harmful, permanent, nicotine delivery vehicle, and I wish them well. -- Trovatore (talk) 08:28, 3 January 2013 (UTC)[reply]
"it's about like caffeine" Spurious speculation is a bad thing Sturat. Perhaps cite more and hypothesize less. — Preceding unsigned comment added by 217.158.236.14 (talk) 08:45, 3 January 2013 (UTC)[reply]
Whoops, wrong number of tildes. Corrected now. I'm not StuRat. --Trovatore (talk) 09:01, 3 January 2013 (UTC)[reply]
The prescription versus nonprescription issue may well be enlightening, if you have the stomach for it. Wnt (talk) 15:05, 3 January 2013 (UTC)[reply]


So.... does anybody know why the inhaler disappeared from over the counter sales in the US? --Jonharley667 (talk) 15:57, 4 January 2013 (UTC)[reply]

Did you not read what we provided? Alansplodge was the first to respond - he confimed with a link that showed that inhalers ARE available in the USA, on precription. The reason why you need a prescription is probably what I said above - such a product would be extremely addictive. And as myself and others have pointed out, medically harmfull -significantly increasing your risk of TIA's, strokes, heart attacks, and other circulatory problems severe in their impact. A dangerous drug in other words - I am surprised it is available at all. Wickwack 124.178.42.57 (talk) 10:00, 5 January 2013 (UTC)[reply]


Did you bother reading what I wrote? It used to be OTC.--Jonharley667 (talk) 00:37, 7 January 2013 (UTC)[reply]

January 3

Hobbitses

feel free to add referenced answers
The following discussion has been closed. Please do not modify it.

This was hatted by Whoop Whoop. I have unhatted it, as while hobbits are indeed fictional characters, such questions are common on Science Desk, and seem a useful way to improve the science knowlege of the OP and others. The discussion on why falling is less haardous for small creatures than it is for large creatures is entirely appropriate on Science Desk Floda 60.228.254.31 (talk) 02:00, 3 January 2013 (UTC) [reply]

A hobbit and a lot of dwarves fall a great distance in 3D but don't go splat. Given that Lemmings require parachutes to avoid splatting, but cats always land on their feet, how what properties do hobbitses and dwarves have to be to avoid going splat? — Preceding unsigned comment added by 31.54.80.187 (talk) 22:34, 2 January 2013 (UTC)[reply]

Being fictional. StuRat (talk) 22:41, 2 January 2013 (UTC)[reply]
As for what enables some animals to survive falls that kill others, mass is the biggest factor. Insects are light enough that they can gently fall, or might even be blown away in the wind. Small rodents can fall quite a distance. Cats and such a bit less. People, less still. An elephant might well die just from falling over.
Secondary to mass is density. Something light and fluffy won't have as high of a terminal velocity as the same mass jammed into a dense sphere (like a falling balled-up armadillo).
Of course having flight surfaces like wings is better yet, but you asked about "falling", and gliding/flying don't really qualify. StuRat (talk) 22:43, 2 January 2013 (UTC)[reply]
Talking about elephants falling over, in certain parts of Africa the marula tree grows. The fruit ripens, drops on the ground, and ferments. It appears elephants then find it delicious, eat it and get falling down drunk (as do some other animals). See http://www.youtube.com/watch?v=EyWd7ozjwbA. No, they don't die - thats' just some typical off the cuff nonsense from StuRat. Elephants can fall over for the usual reasons, though being 4-legged, its a lot less likely than it is for a human. The most common is slipping on mud. See http://www.youtube.com/watch?v=4ENVL37RjC0. Wickwack 120.145.8.219 (talk) 00:41, 3 January 2013 (UTC)[reply]
This site discusses the negatives of using anesthesia in elephants, due to the risk of injury when they fall over: [15]. This site discusses a case where an elephant died as a result of falling into a concrete moat: [16]. While they attribute the severe injuries to it's attempts to climb out, note their statement that: "Elephants cannot convalesce while sitting or lying down because their weight exerts life-threatening pressure on their lungs". Thus, even seemingly minor injuries from a fall can become fatal. StuRat (talk) 01:02, 3 January 2013 (UTC)[reply]
And a typical StuRat comeback seeking to defend an initial error as correct. Your initial post claimed that "an elephant might well die just from falling over", and the context of that post clearly established that you must have meant "from the impact or consequences directly caused by the impact". The references you cited show that elephants are at risk from secondary consequences of unnatural things like falling down man made structures (as it man and just about everything else, and from being unconscious from aneasthesia, as is man and just about everything else. Humans unconscious are at risk of having breathing blocked off - that's why we are taught in first aid training to put them in the coma position. The videos I linked clearly show elephants that have fallen over that, once up again, are no worse off for it. Wickwack 121.221.86.215 (talk) 03:31, 3 January 2013 (UTC)[reply]
I said "might" die, you apparently think "might" means "must". Read a dictionary, please. And the difference, as noted in the link I posted, is that elephants can not heal while lying down or sitting, while people can. You also get something similar with horses, where a broken leg is often fatal, because they can't just lie down until it heals. StuRat (talk) 03:34, 3 January 2013 (UTC)[reply]
So elephants might die from relatively long term laying down - that's not from the impact of falling over. There is indeed a difference between "might" and "must", but either way you are clearly wrong. There's nothing in what I said that implies "must" anyway. If you are shot in the chest, you might die directly from that. But you might not too. One might say a human might die from sneezing. A sneeze can initiate a bloody nose, if there is pre-existing or coincident medical condition, it could result in hemorage or infection and death. But we don't normally go around telling each other sneezing is dangerous. Wickwack 121.221.86.215 (talk) 03:52, 3 January 2013 (UTC)[reply]
What do you mean by "Lemmings require parachutes to avoid splatting"? Nyttend (talk) 00:28, 3 January 2013 (UTC)[reply]
He's referring to an infamous 1950's Walt Disney movie that portrayed lemmings committing suicide en mass by running over a cliff. Lemmings don't do that, but the director thought that the movie was boring and decided to spice it up by having something hidden from camera view throw a large quantity of captured lemmings over the edge. Wickwack 120.145.8.219 (talk) 00:41, 3 January 2013 (UTC)[reply]
Lemmings (video game); [17]. -- Finlay McWalterTalk 00:44, 3 January 2013 (UTC)[reply]
Hobbitses have enormous feet that distribute the impact over a greater surface area (and also provide extra drag on the way down). Dwarves are just really tough, the Chuck Norrises of Middle Earth. Clarityfiend (talk) 01:06, 3 January 2013 (UTC)[reply]
As far as I can recall, the movie showed the falling characters either landing on something somewhat soft, or hitting multiple surfaces on the way down, so no one fall was all that long. Videos of skiers doing that sort of thing while falling down frightfully steep slopes and living to tell about it suggests the movie isn't as much of an exaggeration as it might at first appear. Jc3s5h (talk) 01:13, 3 January 2013 (UTC)[reply]
Sounds fair. I saw the movie once when it was on TV but it was quite a while ago. According to the Wiki article, the voice over said that the leemings were jumping into the sea and were expecting to swim across, and would socumb to exhaustion and drown. However, the movie scene was faked, and it became infamous because it was faked, and the incorrect concept of lemmings committing suicide has entered our culture. Wickwack 60.230.235.129 (talk) 01:26, 3 January 2013 (UTC) [reply]
Are you sure that myth didn't predate the movie ? StuRat (talk) 02:08, 3 January 2013 (UTC) [reply]
It could have. One of the references linked in the Wiki articles on lemmings sort of vaguely implies that the Disney people confused suicidal behavior with mass movement from one teritory to another, crossing streams where necessary, which is what some sorts of lemmings actually do. I recall seeing a TV documentary on animal movie fakery tracing the myth origin to the Disney film, but I cannot recall who made the documentary, and such documentaries are hardly 100% trustworthy anyway. A number of written sources which I don't have to hand right now said that Disney's delibrately spiced up the movie in this way - it is unfortunatly consistent with what Disney did in the 1950's - such as contriving fights between animals that don't even live in the same regions. Wickwack 121.221.86.215 (talk) 02:56, 3 January 2013 (UTC) [reply]

Whoop was right to hat this. μηδείς (talk) 03:25, 3 January 2013 (UTC)[reply]

Name that fish part 2: Green-eyed monster

Mystery fish

Still sorting my vacation photos. Seen on a reef, close offshore in Cuba. This time around, I have no idea what this fish is. As before, once identified feel free to add to any articles that might need it. Thanks, KevinHadley (talk) 02:31, 3 January 2013 (UTC)[reply]

Grouper ? StuRat (talk) 02:33, 3 January 2013 (UTC)[reply]
Shape looks like a Tetraodontidae or Pufferfish. No idea which species. --Jayron32 02:34, 3 January 2013 (UTC)[reply]
Although very difficult to see, this fish does appear to have spines and its markings are very similar to this fish which is a Diodon and of the five species listed, the Long-spine porcupinefish is the closest match. Modocc (talk) 05:27, 3 January 2013 (UTC)[reply]
Thanks to Jayron32 and Modocc. That image does indeed look like...well, a spitting image of my fish. Looks like the Wikipedia article already has a better image, but I'm still quite pleased to have seen one in the wild. Cheers! KevinHadley (talk) 00:10, 4 January 2013 (UTC)[reply]
May we mark this resolved ? StuRat (talk) 00:39, 4 January 2013 (UTC)[reply]

Name that flower

Name that flower

Okay, this is the last one for today, I promise.

This photo was taken in mid-October, in Ontario, Canada, about 100 km north of Toronto. Genus? Species? KevinHadley (talk) 02:42, 3 January 2013 (UTC)[reply]

Chicory? --Jayron32 02:45, 3 January 2013 (UTC)[reply]
Agreed, although this pic is somewhat unusual in that the outer edge of each petal seems to have curled back on itself. StuRat (talk) 03:11, 3 January 2013 (UTC)[reply]
There are several different varieties of chicory/endive with slightly different shapes and colors to the petals, but they all look broadly like this. this search should be convincing enough that the flower is chicory. Several have petals that look like the above. --Jayron32 04:07, 3 January 2013 (UTC)[reply]
My immediate thought there is that the particular flower is on the last legs of it's flower-ness, probably a couple hours (iirc as to how long chicory flowers last) before it starts to look like it's dying. Lsfreak (talk) 03:24, 3 January 2013 (UTC)[reply]
Ah, thanks, Lsfreak (and again to Jayron32). If the curly petals are a sign of a dying flower, then I shan't go shopping for an article for these. They have artistic value, but probably aren't all that educational. KevinHadley (talk) 00:12, 4 January 2013 (UTC)[reply]
It might still be useful. The way in which flowers wilt is also helpful for identification, after all. In any event, can we mark this resolved ? StuRat (talk) 00:34, 4 January 2013 (UTC)[reply]

Is there much worry about corrosion there, if they are only in contact while I'm cooking  ? StuRat (talk) 02:56, 3 January 2013 (UTC)[reply]

No. 24.23.196.85 (talk) 02:59, 3 January 2013 (UTC)[reply]
I try to avoid mixing aluminum and stainless steel when I can; I've ruined quite a number of aluminum utensils and pots when I carelessly rested them against something stainless steel in the dishwasher, for example. You may be OK, but anything either particularly acidic or basic could create an environment that cause galvanic corrosion of the aluminum lid, which can lead to a nasty grey film which never seems to go away. --Jayron32 04:03, 3 January 2013 (UTC)[reply]
I'm not as concerned about the aluminum lid as the stainless steel pot. Will it be OK ? StuRat (talk) 06:24, 3 January 2013 (UTC)[reply]
Yup. Aluminum is a more active metal than iron (see Standard electrode potential (data page), and note that the Al3+ + 3 e -> Al is much more negative than the Fe2+ + 2 e -> Fe) so when in contact aluminum acts as a Sacrificial metal when conditions exist that would normally cause steel to rust. Your steel pot should do nothing at all. Your aluminum lid, is iffy. --Jayron32 07:12, 3 January 2013 (UTC)[reply]
OK, excellent. The lid is old and already looks ugly, while the pot is shiny and new, and I want to keep it looking good. I will mark this Q resolved. StuRat (talk) 18:36, 3 January 2013 (UTC)[reply]
Resolved

Why do you get x2 times the kinetic energy from x times the joules?

That doesn't seem right to me.

Seems like the universe just enjoys explosions and doesn't like high-speed rail. Sagittarian Milky Way (talk) 04:28, 3 January 2013 (UTC)[reply]

What? What makes you think "you get x2 times the kinetic energy" from "x times the Joules"? Work-energy (ΔK=W) says X J of work (if that's what you mean) can be used to impart X J of kinetic energy, not X2. 72.128.82.131 (talk) 05:33, 3 January 2013 (UTC)[reply]
You use 1 joule of energy to move a stationary kilogram in space, you get 1 m/s, for a kinetic energy of 0.5(12). You use 10 times more joules to move another kg, you get 10 m/s for a kinetic energy of 0.5(102), which does 100 times more damage then the first kilo when it hits something, but is only 10 times harder to stop (by applying 10 joules in the opposite direction). Sagittarian Milky Way (talk) 06:01, 3 January 2013 (UTC)[reply]
Your premise is false. You use 100 times more joules (energy) to accelerate 1 kg from 0 to 10 m/s as from 0 to 1 m/s. The energy you put in is equal to the final kinetic energy. — Quondum 06:12, 3 January 2013 (UTC)[reply]
(edit conflict) No SMW, you've confused something horribly. KE = 1/2 m * v2. If you use 1 joule to move 1 kilogram, it moves at a speed of 1.414 m/s √(1/0.5). If you use 10 joules to move 1 kg, it moves at a speed of 4.47 m/s √(10/0.5). I'm not sure what math you're doing, but it is a mess. --Jayron32 06:15, 3 January 2013 (UTC)[reply]
  • You appear to be confusing "Joules" with velocity, because it is indeed correct that you must do x2 times the work to accelerate an object to x times the velocity, starting from rest. The joule is the unit of kinetic energy, however, so I don't believe your question makes sense.--Jasper Deng (talk) 06:52, 3 January 2013 (UTC)[reply]
Forget the x squared, horrible brain fart on my part. Why should a falling object gain 0.5 kinetic energy (0 to 0.5) the first unit of time and 1.5 the next (0.5-2) when the force on it is the same (well an infintesimal amount more from getting closer to the center of the Earth)? Okay, that's just the way the universe is. It makes punches and car accidents many more times more serious than if it was KE=mv, but there doesn't seem to be an elegant reason for why it's this way. Sagittarian Milky Way (talk) 07:04, 3 January 2013 (UTC)[reply]
As far as energy and gravitation, (i.e. falling objects), see Equivalence principle. The acceleration of an object doesn't matter whether it is caused by gravity or by a giant jet engine. Acceleration is acceleration. Also, (at non-relativistic speeds), a force continually increases the amount of kinetic energy an object has, this is fundemental, first semester physics stuff. It doesn't matter whether the force is gravity or the force is a jet engine, applied force increases energy, work = force x distance: the longer distance you apply a force, the more kinetic energy you impart on the object. --Jayron32 06:56, 3 January 2013 (UTC)[reply]
I think what he means is that K=(1/2)mv^2=(1/2)m(at)^2, and then dK/dt=mta^2. However something must be wrong with that math because a constant conservative force must do work at a constant rate as you say and I know.--Jasper Deng (talk) 07:06, 3 January 2013 (UTC)[reply]
Yeah, my calculus is 20 years old, so I'm not sure I can check you on that, but logically a joule is a kilogram ⋅ meter2 ⋅ second-2 while a newton is a kilogram ⋅ meter ⋅ second-2. They're both second order in time, so over time added force should create a linear response in energy. Doubling the applied force should cause a doubling of dK/dt. That's right, isn't it, logically speaking? --Jayron32 07:18, 3 January 2013 (UTC)[reply]
(edit conflict)OK, I now see what's going on. The derivative (i.e. rate of change) of kinetic energy with respect to displacement is constant for a constant force. However, the derivative with respect to time is directly proportional to time. This is actually because although the power (rate of change of kinetic energy w/ respect to time) increases, that is because the force must do the same amount of work over the same distance but in correspondingly less time. --Jasper Deng (talk) 07:23, 3 January 2013 (UTC)[reply]
...which is a direct consequence of the fact that the integral of a linear function is a quadratic function. So, if a force is constant, then the integral of force over distance is linearly increasing; and because force induces acceleration according to a law of constant proportionality (at least, to a very good approximation in many cases), the increasing velocity is the solution to a second-order integral of a constant function. When formulated this way, it is almost self-evident that the parabola should show up so many places in nature: it's among the most simple consequences of the application of mathematical law to the most extremely simple of all natural phenomena. This is discussed at great length in Isaac Newton's work on the mathematical principals of nature. We can thoroughly understand how a double-integral of a constant function yields a parabola. We can show how a constant function well-approximates the force due to gravitation, at least over a short distance near the Earth's surface. And therefore, we can show how a simple set of rules leads to a squared term with respect to velocity. If you wish to know why these things are so, neither I nor Sir Isaac frame any hypotheses. Nimur (talk) 08:25, 3 January 2013 (UTC)[reply]

Facial recognition after many years

When I watched the CSI episode "Living Legend," something in that episode puzzled me: Catherine Willows had a chance encounter with Mickey Dunn when she was 16, but the two would not see each other again for another 30 years, when Dunn is arrested after returning to Las Vegas. In the interrogation room, as Willows started to head out after making a brief reference to her earlier encounter with Dunn, Dunn immediately identified her as "Sam Braun's daughter," and stated "You think that I would ever forget a face? That face?" Does this make scientific sense? Is it really possible for someone to recognize a person by their face at age 46 based on a chance encounter with that person at age 16 (and no previous or further encounters), or is this a bit too exaggerated? 24.47.141.254 (talk) 06:04, 3 January 2013 (UTC)[reply]

I think the specific "chance encounter" would have to be clarified, as some chance encounters would leave lasting memories while other chance encounters would not leave lasting memories. Bus stop (talk) 06:07, 3 January 2013 (UTC)[reply]
I suspect that people's ability to recognize faces varies dramatically. So, while most people probably couldn't do that, maybe some can. StuRat (talk) 06:22, 3 January 2013 (UTC)[reply]
This is my reaction as well. Facial change from 16 to 46 is not always dramatic, meaning that many people should be able to recognise someone they knew from the earlier age, especially if they've aged less than most. I have known some people to recognise people under far worse conditions. There are many cues aside from simple facial recognition that people use, including facial expressions, mannerisms, speech characteristics, body movement, etc. It takes one atypical gesture to bring back a memory of someone who previously used that gesture. Add to that a person with an exceptional memory and ability to pattern match from the cues, who might have been struck by the person at the time and thus observed and memorised them more acutely than they were admitting to, and this is not implausible in a plot. I routinely see strategems used by TV serial scriptwriters that I find far less plausible than the one you describe. — Quondum 08:38, 3 January 2013 (UTC)[reply]
It's possible if you combine little change in one person with good memory in the other. <original research>Some years ago, I was at a store with my mother when someone began calling her by her maiden name — it was a high school classmate, and they'd not seen each other since they were slightly older than 16; at this time they were about 46 or slightly older.</original research> Nyttend (talk) 15:03, 3 January 2013 (UTC)[reply]
Purely anecdotal evidence, but I attended a junior school reunion a couple of years ago. Many of us had not seen each other since the age of 11 and were instantly recognisable. Others had changed beyond recognition and had to to tell us their names. Alansplodge (talk) 18:10, 3 January 2013 (UTC)[reply]
Also, since girls mature faster, the chances of recognizing a woman at 46 you hadn't seen in 30 years are probably better than for a man, since males still tend to change appearance quite a bit after age 16, before stabilizing into their adult look. However, more women change their looks intentionally, by changing hair color and style and having nose jobs, etc. StuRat (talk) 18:32, 3 January 2013 (UTC)[reply]

Taj Mahal question

two ways of protecting the tajmahal from air pollution. — Preceding unsigned comment added by 78.101.62.16 (talk) 06:33, 3 January 2013 (UTC)[reply]

Maybe you could do what looks like your own homework by looking up what forms of pollution cause damage to marble and how their impacts can be minimised. HiLo48 (talk) 07:28, 3 January 2013 (UTC)[reply]
1) Seal it in a large hermetically sealed crypt.
2) Transport it to the moon, where there is no pollution.
Hope this gives you some ideas. — Preceding unsigned comment added by 217.158.236.14 (talk) 08:58, 3 January 2013 (UTC)[reply]
I'm not sure that the second solution would work - if the Taj Mahal were on the moon, wouldn't it end up pock-marked with moon craters of various sizes faster than it would on the Earth, due to lack of atmosphere there? --Demiurge1000 (talk) 19:16, 3 January 2013 (UTC)[reply]
Yes, but it's not as fast as you might think. Many of the tiny craters on the Moon are millions of years old. With no erosion, they stick around for a long time. StuRat (talk) 00:45, 4 January 2013 (UTC)[reply]
(ec) Sure, it would end up eroded by meteor impacts faster than it would on Earth—but 'faster' isn't he same as 'fast'. I do wonder what the time scale of that erosion actually is, and how it compares to the effects of air pollution (or, for that matter, plain old erosion by wind and rain) on Earth. This paper estimates that there would be a bit less than one new one-millimeter crater formed per square meter of exposed surface, per decade. Sure, one could get really unlucky and catch a larger rock that destroyed the whole building, but the odds are very much against it. TenOfAllTrades(talk) 01:00, 4 January 2013 (UTC)[reply]
Since this does sound a lot like a homework question - I'm not going to give you the answer directly - but instead offer some help in finding the answer yourself. This is an interesting question, not because the answer is interesting - but because of what you have to learn in order to find possible answers.
For that reason, you need to read the following articles: Taj Mahal (specifically, the section: Taj_mahal#Threats) and also White marble (which is what the Taj is mostly made from). Then look at Acid rain - and follow links from the White marble article to calcium carbonate and calcium magnesium carbonate (constituents of white marble). Pay special attention to calcium carbonate#Chemistry where it discusses what happens when acid meets up with it. Taj_mahal#Threats also mentions the Mathura Refinery - and reading that will tell you much more about where the acid rain is coming from and why it's about to get a whole lot worse if someone doesn't intervene to prevent that.
Then, the other problem described in Taj_mahal#Threats relates to the water levels in the Yamuna river - so read that article to figure out why the level of water in that river is dropping so rapidly. Ask yourself where the water is going. Is it being consumed, dammed or diverted somewhere up-stream from the Taj - or is the flow rate of the river being reduced because of less rainfall - could it be that climate change is involved here? Maybe there is more than one reason.
Once you understand the reasons why the Taj Mahal is in so much trouble - you should think carefully about whether it would be possible to add protection to it (Could you coat the marble with something to keep out the acid rain and do something shore up the disintegrating foundations?) or whether you could reduce the problems that beset it (Cut out some of the pollution and solve the problems of the Yamuna river?)...which will lead you to more interesting articles in the largest repository of all human knowledge...Wikipedia. That should give you at least two possible ways to save the building.
Having read all of those things in order to provide you with the articles that will answer your question, I don't think things will end up well for the Taj Mahal. However, India should realize that it's like the canary in a coal mine - the Taj is only the most publicly and internationally noticeable symptom of the general state of environmental problems there. If you can't save something as important artistically, culturally (and tourism dollar raising!) as the Taj Mahal - how can you possibly hope to keep a reasonable standard of air and water quality elsewhere in the country?
SteveBaker (talk) 14:04, 3 January 2013 (UTC)[reply]


Shrink wrap. ←Baseball Bugs What's up, Doc? carrots00:41, 4 January 2013 (UTC)[reply]
Give Christo a call. -- Jack of Oz [Talk] 00:47, 4 January 2013 (UTC)[reply]
Human extinction. - ¡Ouch! (hurt me / more pain) 18:14, 5 January 2013 (UTC)[reply]

Clean big city water supplies

I live on the outskirts of Melbourne, Australia, a city with over 4 million residents. Yesterday I went up into the nearby mountains where a lot of the city's water catchments are, and saw some of the signs I've been seeing all my life telling me that (as our supplier's website tells me)...

Melbourne has some of the cleanest drinking water in the world. What falls in the water catchments is pretty close to what comes out of our taps.
Any activities which could contaminate or pollute the water are not allowed in our catchments and reservoirs....

This is all very nice, and it's true that the catchments are pretty well locked up, but it got me wondering. Is the claim of "some of the cleanest drinking water in the world" a valid one? Has anyone really done the comparison? Who else has got clean water? HiLo48 (talk) 07:24, 3 January 2013 (UTC)[reply]

New York City. --NorwegianBlue talk 14:53, 3 January 2013 (UTC)[reply]
But its all based on weasel words, "some of the cleanest...", "world class", what do they mean, on which scale (not the lime scale!), compared to what. If any editor included those phrases without a ref in WP they would get a warning template muy pronto. Advertising bull***t. Richard Avery (talk) 15:35, 3 January 2013 (UTC)[reply]
Not necessarily cleanest but they have contests for best-tasting tap water: [18] Rmhermen (talk) 16:01, 3 January 2013 (UTC)[reply]
Australia, being rather isolated geographically, is far from most of the world's sources of particulate air pollution (especially in the prevailing wind direction). So, in that sense, the rainwater there is likely relatively clean. Places with cleaner precipitation, like the South Pole, don't collect it on a large scale like that. StuRat (talk) 18:26, 3 January 2013 (UTC)[reply]
Australian cities do not in general have particularly good water supplies, by Western standards. One never hears much about water quality in Melbourne, but Sydney has over recent years often been in the news media due to unacceptable water quality. StuRat is correct in his point about pollution, but he has overlooked that Australia is a dry country forcing us to use water sources that other countries would avoid. (Most locations have water rationing - by law I can only water my garden in twilight hours on Wednesdays and Saturdays.) Perth gets up to 30% or so of it's water from underground resevours - this water requires considerable treatment to get it drinkable, and as delivered at your tap, is not particularly good. Some Australian cities are in the next few years are going to begin recycling sewage water into drinking water supplies as that is the only water they can meet the needs of a growing population at low cost - the risks are hard to quantify and probably significant, but they are going to do it anyway. Sea water desalination has been implemented, but it just too darn expensive in energy requirements and the saline discharge adversely affects marine life, forcing shutdowns. The West Aust State Health department some years ago put out a pamphlet encouraging rainwater tanks for better health, but as the Water Authority is another State Govt body, a bun fight ensued and the Health Dept got told to pull their head in.
We look with envy at water qunatity and quality in the USA. The USA uses ultraviolet treatment to kill microbes. As far as I know, this is not done in Australia for municipal water for houses etc, its only used in the better private hospitals.
The New York pamphlet provided by NorwegionBlue is actually very good. The claim about weasel words is not relavent because in the last page it has a detailed technical analysis. What we need is someone to post similar analysises for Australia and other countries.
Wickwack 60.230.194.70 (talk) 00:05, 4 January 2013 (UTC)[reply]
With regards to the USA and ultraviolet, the USA has thousands of water districts and millions of private wells, so any treatment you can think of is probably used somewhere. Chlorination is the most popular antimicrobial treatment, simply because it has the longest history and is best-understood, but you can also find public water supplies that use ultraviolet, ozone, micropore filters, or even no treatment at all (deep aquifers often have no dangerous microbes in them). --Carnildo (talk) 02:09, 4 January 2013 (UTC)[reply]
Have done some digging. Water suppliers in Australia are supposed to comply with Australian Drinking Water Guidelines (ADWG) 2004 - a national standard. There is an exemption granting process for where it is considered not realistic or necessary to meet the guidelines. For example, supply to a number of towns in Western Australia have been granted exemptions, and in these cases the water is considered acceptable for adults but is not safe for infants. I have reviewed the fine print in the service contract for my water supply (from the WA Water Corporation - a Govt owned organisation) and it specifically states that while the Water Corporation treats its water to ensure it meets health and quality guidelines, the householder is responsible for ensuring that water he/they consume is of what is deemed adequate quality. In other words, worse than weasel words - if you get sick from the water, its' your problem. There is a large quantity of info at http://watercorporation.com.au/P/publications_water_quality.cfm. Wickwack 58.170.170.50 (talk) 02:58, 5 January 2013 (UTC)[reply]

Why doesn't color-safe bleach (or bleach in general) oxidize the material of cotton clothes ?

Cellulose, a component of cotton, seems vulnerable to oxidation because of the presence of activating hydroxyl groups. Does the polymer nature of cellulose sterically hinder the oxidation of cellulose by hydrogen peroxide or sodium hypochlorite? 72.229.155.79 (talk) 08:46, 3 January 2013 (UTC)[reply]

The C-OH like you find it in methanol H3C-OH is very stable and neither hydrogen peroxide or sodium hypochlorite can oxidize it at room temperature. The aldehyd group is much more reactive against oxidation and would react quickly, but all relevant CHO groups are bound to the next glucose unit in the cellulose chain. The good thing with dye on cellulose is that the fibre is hard to penetrate and the OH groups make it hard for the more hydrophobe groups to attach. So the dye is easy to reach for the bleach and the bleaching is over before the cellulose is attacked. --Stone (talk) 11:18, 3 January 2013 (UTC)[reply]

Solving linear equation

Given a set of linear equation AX=B, say A is an ill posed matrix (has a few singular value equal very close to zero), which numerical algorithm (conjugate gradient, least squared or steepest decent etc ) should be used to obtain the best solution? More specifically, is there a concrete comparison between these methods? — Preceding unsigned comment added by 210.212.187.69 (talk) 09:06, 3 January 2013 (UTC)[reply]

The Mathematics desk is this way... Roger (talk) 11:15, 3 January 2013 (UTC)[reply]

Inhaling baby powder

What happends if you inhale babby powder? — Preceding unsigned comment added by 120.148.21.19 (talk) 13:35, 3 January 2013 (UTC)[reply]

We can't give medical advice. If you are concerned, contact a physician or a poison control center. In the US the number is 1-800-222-1222. 209.131.76.183 (talk) 13:40, 3 January 2013 (UTC)[reply]

what happends if you inhale babby powder? — Preceding unsigned comment added by 120.148.21.19 (talk) 13:43, 3 January 2013 (UTC)[reply]

I see what you did there. I think the answer depends on how the babby powder formed. AlexTiefling (talk) 14:18, 3 January 2013 (UTC)[reply]
Are you the one who asked "how a babby is formed?" a few weeks ago? If so, please stop with this tired meme. We aren't interested. If you are serious, ring the number listed above. — Preceding unsigned comment added by 217.158.236.14 (talk) 14:48, 3 January 2013 (UTC)[reply]
(WP:AGF) Our article Baby powder explains this, right there in the first paragraph: "Talcum powder is harmful if inhaled since it may cause aspiration pneumonia or granuloma." Only some brands are made from talcum powder - others use cornstarch precisely to avoid this inhalation problem. It also explains that "Drugs such as cocaine are sometimes cut with talcum powder." - which is presumably why inhalation is such a big concern. SteveBaker (talk) 14:52, 3 January 2013 (UTC)[reply]
Note the chemical similarity between talc and certain types of asbestos. As explained in the article, talc is not widely believed to be a carcinogen when it is free of asbestos-like fibers. But how sure are you that anyone ever really checked the talc in your bottle is free of those? Note also that talc can be free of "asbestos" fibers but contain "transitional" asbestos-like fibers that do not violate regulations but whose safety is not so clear. [19] Wnt (talk) 15:13, 3 January 2013 (UTC)[reply]
Side discussion about medical advice questions.
I suspected trolling when I saw the babby spelling, but I figured it was simple enough to brush it off with a mention of no medical advice. 209.131.76.183 (talk) 15:24, 3 January 2013 (UTC)[reply]
No matter how much you don't like a question, you should NEVER "brush it off with a mention of no medical advice". That's a cop out. That's an arrogant, insulting approach. HiLo48 (talk) 17:22, 3 January 2013 (UTC)[reply]
What should I have done? If the question is serious, then we can't answer it because it is medical advice. By "brush it off..." I meant that I simply told them we can't answer the question without mentioning my suspicion of trolling - I hoped the quick "we can't do that, contact a doctor" answer would stop the thread without really giving a troll a chance to respond (defending against accusations of trolling/adding pointless but attention-generating follow ups) and draw things out. 209.131.76.183 (talk) 18:39, 3 January 2013 (UTC)[reply]
Sorry if the "What should I have done?" sounds confrontational - it is an honest question. I'd like to handle things correctly here, and even if you don't like my reasoning for why I answered how I did, I feel that I gave a proper response. 209.131.76.183 (talk) 18:48, 3 January 2013 (UTC)[reply]
The rule of thumb to decide whether a questioner is asking for medical advice, by more-or-less consensus after countless discussions on Wikipedia_talk:Reference desk, is Kainaw's criterion: Can the question be answered completely without providing a diagnosis, prognosis, or treatment advice?. In this case, there was no question about diagnosis or treatment advice. One could argue that the questioner in this case asked for a prognosis after a hypothetical inhalation of "babby powder". However, my understanding of the prognosis part of Kainaw's criterion, is that it applies to the prognosis for a real person (such as the questioner or someone they know), and not the prognosis for a hypothetical person after a hypothetical exposure to a possibly harmful substance. I would therefore regard this as a question about medical facts (albeit a silly one), and not a request for medical advice. Since a smart questioner who needs medical advice could camouflage the question as a question seeking facts only, a warning about contacting a poison control center or a doctor if the question relates to a real-life situation could be appropriate. But there is IMO no reason to reject the question in this case. --NorwegianBlue talk 19:44, 3 January 2013 (UTC)[reply]
This debate about how to answer a question belongs on the Talk: page - not here. SteveBaker (talk) 19:51, 3 January 2013 (UTC)[reply]
Agreed. However, this topic has been discussed there ad nauseam. I'll just collapse the side discussion for now. If anyone wants to move it, feel free! --NorwegianBlue talk 20:36, 3 January 2013 (UTC)[reply]

Cleaning a fish tank

This is really nice (I'd jump through it because it's like 40 minutes long), but how would such a fishtank be cleaned? Any gravel siphoning would destroy the aquascaped design? DRosenbach (Talk | Contribs) 18:02, 3 January 2013 (UTC)[reply]

Ideally an aquarium is naturally in balance such that the organisms in it clean the tank itself. Catfish are good for this, for example. This isn't always possible, of course. Filtering the water as it's pumped also can clean it, somewhat. StuRat (talk) 18:18, 3 January 2013 (UTC)[reply]
With an aquarium like this, regular water changes would be done to keep the levels of nitrites->nitrates down. This inhibits algae. Species like Amano shrimp can be introduced to help keep plants clean. I've had tanks similar to this. They require good bacterial filtration (external) as well as regular water changes and a lot of maintenance. If you want to see some amazing examples of tanks like this, search for "Amano tank". Zzubnik (talk) 09:44, 4 January 2013 (UTC)[reply]

Is this aircraft an A330?

In File:Delta Air Lines Boeing 747-400 KvW.jpg I suspect the Air France jet in the back is an A330 series. Would anyone tell me if this is correct? WhisperToMe (talk) 19:06, 3 January 2013 (UTC)[reply]

I really don't know my airplanes (pardon me, [[fixed-wing aircraft), but from a Google image search, Air France A330s seem to have, um... wing tips? The tips of the wings folded up? There's probably a technical term for that. But the plane in that picture doesn't seem to have that feature. --BDD (talk) 20:23, 3 January 2013 (UTC)[reply]
(It's called a winglet.)...and they are indeed standard features on the A330 - so I think this must be an older model than that. Winglets were only a retrofit option on the A320 - so maybe that's what this is? SteveBaker (talk) 20:40, 3 January 2013 (UTC)[reply]

The aircraft in the background of your linked picture appears to be a Boeing 777-200. The shape of the tail and the triple wheeled bogeys give it away. http://en.wikipedia.org/wiki/777-200#777-200. Happymulletuk (talk) 22:26, 3 January 2013 (UTC)[reply]

What Happymulletuk says, it's a 777: an A330 has a round tailcone with an APU exhaust and four-wheel bogies [20]. Acroterion (talk) 22:31, 3 January 2013 (UTC)[reply]

Alcohol and illness

This is just a request for some science, not medical advice. What effect do alcoholic beverages have on everyday sicknesses, such as the common cold or flu? Folk wisdom is always to drink plenty of fluids, so why not some beer and wine? Probably because alcohol dehydrates, and the point of those fluids is to stay hydrated. But alcohol is a proven disinfectant, so it doesn't seem wildly implausible that ingesting it could kill some internal pathogens. So my question is not "Should I drink booze when I'm sick?" It's more like "What effect does booze have on sick people?" --BDD (talk) 20:21, 3 January 2013 (UTC)[reply]

There is to my knowledge no evidence that ethanol has any therapeutic effect against virus infections such as the common cold or influenza. A pubmed search returned one paper where the authors suggested that wine intake may have a protective effect against common cold (prophylactically, not therapeutically): PMID 11978590. Another study concluded that acute intake of moderate amounts of red wine or alcohol had no effect on the immune system of healthy men: PMID 12474070. There is evidence that a moderate intake of alcoholic beverages (especially red wine) may have a protective effect against cardiovascular disease and certain types of cancer: PMID 22852062, but it is not clear whether the effect is caused by the ethanol or other constituents of the beverage. Finally, ethanol may be used as an antidote against methanol poisoning: PMID 21740138.
See also:
--NorwegianBlue talk 21:07, 3 January 2013 (UTC)[reply]
The thing about colds and flu is that they are viral diseases - not bacterial. Viruses spend a good deal of time inside your cells, making more viruses - but they are just DNA, which makes them extremely hard to eradicate without also killing your own DNA. Alcohol has no effect whatever on them once they are inside your body.
To be completely clear (and according to common cold):
  • There is no medical treatment whatever that will prevent you from getting a cold.
  • There is no cure whatever for a cold.
  • Once you have the cold, there is no proven way to reduce the duration of it.
  • Very few "treatments" even help to relieve symptoms more than placebo.
  • Frequent hand washing will help you to avoid getting a cold. There is no evidence that particular soap additives improve hand washing effectiveness.
  • People are asymptomatic for a while after getting an infection - but they are still spreading the disease everywhere - so simply staying away from obviously sick people only helps to a small degree.
  • Wearing a face mask may help you to avoid catching a cold.
  • There is no innoculation against the disease because it evolves too quickly and there are far too many strains of it out there (and no, contrary to what some people claim, flu shots only help you to avoid influenza - not the common cold).
  • Ibuprofen, acetaminophen. paracetamol, etc will get your temperature down - which may make you feel a little better - but because your body is pushing up your temperature in order to help it fight off the virus, it's not going to help you rid yourself of the virus any faster - and arguably might slow that process down, thereby prolonging the cold.
  • Anti-histamines and decongestants may help you breathe a little more easily - but most of the truly effective ones are no longer available over-the-counter in many countries - and the replacements fare no better than placebo. However, some will make you sleepy - and you might prefer sleeping to being away while sick with a cold.
  • Same deal with cough medicines - most are no better than placebo...the good ones are prescription only.
  • There is no evidence whatever that increased fluid intake either improves symptoms or shortens the course of the cold.
  • Humidified air, alcohol and honey are all no better than placebo...same deal for herbal, homeopathic medicines and stuff like reflexology and the like.
  • Antibiotics might help if you get secondary infections (like ear-aches) - but they don't affect viruses - so they don't help your cold at all.
  • Zinc, echinacia and vitamin C might be helpful - but evidence from scientific research is patchy. If there is an effect, it's a very small one. Zinc is bad for you in larger doses - so you should probably avoid that one. However, taking vitamin C BEFORE you get the cold might help reduce it's severity...but again, the research is patchy and the statistical reduction isn't more than a few hours out of a week-long cold...and by the time you're getting symptoms, it's too late.
  • Just about every cold cure/treatment/preventative out there is junk - don't waste your money on them.
Bottom line is that we're totally helpless here. Nothing you do will completely prevent you from getting a cold, nothing will cure it, nothing will shorten the duration. Only the very obvious general pain killers will relieve your symptoms to any significant degree.
The common cold is a really, REALLY tough disease to deal with. SteveBaker (talk) 21:16, 3 January 2013 (UTC)[reply]
Not just zinc, echinacia, and vitamin C—chicken soup also reduces the severity of a cold. Whoop whoop pull up Bitching Betty | Averted crashes 02:38, 4 January 2013 (UTC)[reply]
SteveBaker - that's a great post. It combines a lot of stuff I mostly knew, but you said it all so clearly and firmly.

Thank you. HiLo48 (talk) 21:23, 3 January 2013 (UTC)[reply]

If you put pure spirits on a bacterial infection on your skin, it can help, but it might sting a lot, and you won't really be enjoying a drink. HiLo48 (talk) 21:39, 3 January 2013 (UTC)[reply]
It has been suggested that the beneficial effect of (a moderate intake of) alcohol on the risk of getting a myocardial infarction could be a consequence of its bactericidal effect on oral bacteria: PMID 22998953, see also PMID 19040579. There is evidence that moderate alcohol consumption facilitates the elimination of Helicobacter pylori: PMID 20123162. Finally, excessive alcohol intake increases the risk of acquiring gonorrhea (PMID 9858351) and other sexually transmitted diseases. --NorwegianBlue talk 22:01, 3 January 2013 (UTC)[reply]
I suspect the latter has more to do with behaviors while under the influence of alcohol, but thanks, this is good. --BDD (talk) 22:39, 3 January 2013 (UTC)[reply]
Yep, see beer goggles. :-) StuRat (talk) 22:51, 3 January 2013 (UTC) [reply]
If you want the disinfectant effects of ethyl alcohol without the harmful effects, try gargling with it and spitting it out. Then again, why not save some money and use mouthwash ? StuRat (talk) 22:04, 3 January 2013 (UTC)[reply]
Even better, (except for the savings), is what a colleague of mine did on at trip to India recently. He brought a large bottle of Aquavit and used it for gargling, thus reducing oral bacterial flora. He then swallowed it, as a prophylactic against Helicobacter pylori in his stomach. Finally, the mildly sedative effect gave him a good night's sleep until the next morning. Hey, but that's three good things all in one - a gargle, a Helicobacterfree stomach, and a surprisingly good nights sleep. --NorwegianBlue talk 00:06, 4 January 2013 (UTC)[reply]
Alcohol is a surface disinfectant. When consumed it is well under 1% by volume of a person's blood. μηδείς (talk) 04:07, 4 January 2013 (UTC)[reply]
If it's obnoxious enough to kill bacteria, and especially viruses, it's probably going to kill your cells too. That's why antibiotics and antiviral medication are so amazing (maybe pesticides too). They interfere with very specific bacterial processes (see gram negative bacteria, for example), often cell membrane construction that don't apply to our cells. Besides, the immune system is ultimately what defeats almost all of these diseases, and alcohol probably slows it down to some extent (see the links above). Shadowjams (talk) 11:51, 4 January 2013 (UTC)[reply]
Well, your cells are protected by a layer of mucus, so the concentration at the live cells is hopefully low enough to prevent them from being killed. And those meds cause resistant strains to develop, so should only be used when absolutely necessary. StuRat (talk) 06:03, 5 January 2013 (UTC)[reply]

I don't understand this. If the universe is only 13.75 billion years old. Then how can we possible see something that further than 13.75 billion years light away up to 40 billion years light? Anything further than 14 billion light years would indicate light didn't have enough time to travel from that far to Earth since it would takes a lot more time for it to get to the Earth. 174.20.15.246 (talk) 03:16, 4 January 2013 (UTC)[reply]

(First), we can see in two directions, and (second) whatever we can see in one direction that is 13.75 billion years old has also moved some 6 million additional years away from us. So the visible universe is 13.75 plus 6-plus billion years times two across. 04:02, 4 January 2013 (UTC)
I already accounted for it, base from the article. The observation from any direction would be 90 billion light years from Earth. How is that possible?174.20.15.246 (talk) 04:15, 4 January 2013 (UTC)[reply]
It's been a while since I've looked at astronomy, but if I remember correctly, this is because the universe was smaller when the light was emitted, and as the light travelled, the universe expanded, increasing the distance between the source and you (the eventual viewer of the light), so by the time the light reaches you, it's travelled a much greater distance than when it first left; in fact, some sources may even have moved so far away that they are more than 14 billion ly away (meaning that if the universe stopped expanding at this moment, and the distance was kept constant, it would take whatever that amount of time to get here). To use an analogy: if you have a deflated balloon and point A and B, with an original distance as 5cm, and an ant (our "light particle/photon) starts travelling from A to B, but you then expand the balloon as the ant walks. The ant is still walking towards B, but the distance between the two is increasing, so that by the time the ant reaches B, the distance may no longer be 5 cm, but rather something like 100cm (it's a huge balloon, let's say). Now look at the amount of time the ant spent walking. If we stopped the balloon's expansion, and had another ant walking at the exact same speed from Point A towards Point B, after the same amount of time, that ant would not have reached point B like the first ant did. This is the same way that a light particle from a 40 Billion ly distant star could now not have reached us in the amount of time, but because the universe was smaller and the source closer when the light was emitted, the particle did reach us (although this travelling over stretching spacetime distorts lightwaves, if I remember correctly, which is what causes redshift). Brambleclawx 04:17, 4 January 2013 (UTC)[reply]
OK according to my understanding, let put it this way. If we see something 13 billions years light away, that means it was 13 billions years light from Earth 13 billion years ago or billions years ago right? So its "current location" right now is much further away due to the expansion of the universe. So right now it could be at 90 billion years light away but apparently we can't see its current location rather what we see is its past. I guess their definition of what is observation is different from mine. So let me ask a question: is it possible for us to see something that is like 20 billion lights years away at its current location right now?174.20.15.246 (talk) 04:55, 4 January 2013 (UTC)[reply]
I wouldn't say it was exactly 13B ly away when the light was emitted. You could probably accurately say it travelled for 13 billion years, but the distance would be relative; in most cases, you would need to look at the rate of expansion of the universe to know the exact distance it travelled. But the idea seems more or less right. For the second question, no. If it's 20 billion light years away, to see it in it's exact location right now, we'd have to perceive the photon emitted at this instant, which will not arrive here until at least 20 billion years (but longer because of the universe's expansion) because 1 lightyear is the distance light can travel in 1 year; to see it right now, the light would somehow have to cover a distance it normally takes 20 billion years to traverse in an instant. Brambleclawx 05:33, 4 January 2013 (UTC)[reply]
Well to keep things simple, I just make some assumption to say it is 13b ly when the light was emitted but I also mention other possibility when I said "billions years ago right?". I understand it now but I got to say that the definition of observable universe is pretty hard to understand especially for layperson or even someone with some knowledge of physic. This makes me think of how much "limited we are from the real universe", everything we are seeing right now are all the past! It's funny when I think about this so let say I see a person stands next to me but what I see is just an image that was carried by light in an instant so that image technically is not exactly the same at the moment when I see it but sometime in the past, very small though, like .00000...1 second ago. No matter where we look at the universe, we always look in the past but the difference the past here can ranging from few seconds ago to billion years ago.174.20.15.246 (talk) 06:00, 4 January 2013 (UTC)[reply]
That's essentially right, but the chances of it being defined as actually "looking into the past" tend to decrease in proportion with the increasing knowledge of the person talking. Special relativity made it fashionable to eschew any notion of absolute time. (And please don't edit others' comments; "travelled" is UK spelling.) Evanh2008 (talk|contribs) 06:25, 4 January 2013 (UTC) [reply]
With that, I would like to thank StuRat for fixing the formatting error in my post above. Irony is many-splendored thing. :) Evanh2008 (talk|contribs) 06:44, 4 January 2013 (UTC)[reply]
This has been discussed before. Someone correct me if I'm wrong, but we can't see anything 40 billion light years away. The universe is bigger than 13.7 billion light years because the speed of light is with respect to space, and space has been expanding rapidly. It cah expand faster than the speed of light. Bubba73 You talkin' to me? 07:19, 4 January 2013 (UTC)[reply]
We can see things that "now" (if such a concept could be defined) are more than 16 billion light years away, but the light that we are now perceiving from those objects began its journey toward us when the objects were significantly closer (less than 16 billion light years away). Evanh2008 (talk|contribs) 07:27, 4 January 2013 (UTC)[reply]

January 4

Microscope image identification

The above images were photographed through the lens of an optical light microscope at 400x magnification. Both are water samples originating from a stormwater management facility (aka storm pond) in Richmond Hill, Ontario, Canada. The second sample was boiled in a glass beaker on a hot plate (reaching a temperature of 94 degress Celsius for 3 minutes). My questions are: is the first one, as I suspect, a Navicula diatom, or some other organism? And the second question is what are the crystals in the second image? It only appeared in water sample which were boiled (appearing as a thin, milky coloured film on the water surface), which led me to rule out beaker contamination, so I hypothesized that it was crystalized cellular debris from lysed cells; am I right, or are the crystals of another origin, such as dissolved minerals crystallized by heat? Your answers are greatly appreciated, Brambleclawx 03:56, 4 January 2013 (UTC)[reply]

Boiled at 94°C ? Are you sure you don't mean the water was evaporated at that temperature ? StuRat (talk) 06:39, 4 January 2013 (UTC)[reply]
Let me correct myself, I tried to boil the water, but it only reached 94°C and did not rise beyond that point. So more correctly phrased, the water was heated to 94°C for 3 minutes. Brambleclawx 17:23, 4 January 2013 (UTC)[reply]
Without knowing the chemical composition of the water, I'd guess that it's the dissolved minerals crystallising out of solution.--TammyMoet (talk) 10:21, 4 January 2013 (UTC)[reply]
So heat can cause minerals to crystallize? Brambleclawx 17:23, 4 January 2013 (UTC)[reply]
See Precipitation. Matt Deres (talk) 18:26, 4 January 2013 (UTC)[reply]
I know what precipitation is, thank you. The article says nothing about heat causing precipitation; it's a chemical reaction that causes precipitation. Are you implying that the heat catalyzed a reaction? Brambleclawx 20:19, 4 January 2013 (UTC)[reply]
Well, the heat causes the water to evaporate, increasing the concentration of the solutes to the point where they precipitate out of solution. StuRat (talk) 05:54, 5 January 2013 (UTC)[reply]
And likely precipitates from water are calcium carbonate or calcium sulfate. Graeme Bartlett (talk) 12:22, 5 January 2013 (UTC)[reply]

Electromagnetic wave

Can a proton emit photon? When I read about photon, it only mentions that photon is emit or absorb by electron.174.20.15.246 (talk) 04:02, 4 January 2013 (UTC)[reply]

According to proton decay, a decaying proton would create a positron and a pion; the pion would then decay into two gamma ray photons. So if proton decay were to occur, then, yes, protons could emit photons. However, that's the thing: proton decay is as of now only a theory. There is no experimental evidence of proton decay ever happening, so it's not clear whether proton decay does occur. So while a theory states it is possible, in real life, there is currently no evidence that protons emit photons ever. Brambleclawx 04:23, 4 January 2013 (UTC)[reply]
Yes, a proton emits a photon whenever it moves - and even when it isn't moving! In modern physics, we actually describe the proton as continuously emitting a sea of virtual photons; some of these convert to real photons when they interact with other particles. Nimur (talk) 04:33, 4 January 2013 (UTC)[reply]
If it is just a theory so far then I just go with it can't emit photon. How about can photon absorb photon? In other word, what would happen if a photon hits a proton? Would it just pass right through the proton? Or would it get absorbed by proton?174.20.15.246 (talk) 04:34, 4 January 2013 (UTC)[reply]
According to the article http://en.wikipedia.org/wiki/%C4%8Cerenkov_radiation, emmision or absorption of a photon can be done by any charged particle (including therefore protons) that is accelerated or decellerated, respectively - the energy has to go/come from somewhere. Wickwack 60.230.207.240 (talk) 04:40, 4 January 2013 (UTC)[reply]
The energy does not have to go/come from somewhere if it is virtual photons we are speaking of (which is the case when like charges repel - protons exchange virtual photons which carry the Coulomb force), it just can't be around for too long. Heisenberg's uncertainty principle states that the conservation of energy may be violated to at most ΔE in a short enough time interval Δt such that ΔEΔt≤ħ/2. (Δ technically means uncertainty - we are assuming that our real observations of both are 0) 72.128.82.131 (talk) 20:26, 4 January 2013 (UTC)[reply]
So would it require certain situation that proton would absorb and photon when photon hits proton? In other words, do proton "always" absorb photon when being hit?174.20.15.246 (talk) 04:58, 4 January 2013 (UTC)[reply]
Inelastic Compton scattering only changes the photon's energy so although it imparts momentum to the proton the photon is not absorbed. --Trillianthcircuit (talk) 07:25, 4 January 2013 (UTC)[reply]
Seas of virtual photons being continuously emitting seems really vague. Are these carriers actually enumerated or are they just an assumed part of the wave function that maps the near field? --Trillianthcircuit (talk) 07:25, 4 January 2013 (UTC)[reply]
If you'd like less vagueness, consider reading the mathematical article I linked earlier this week. Nimur (talk) 07:36, 4 January 2013 (UTC)[reply]

In classical physics, anytime a charged particle is accelerated, it will radiate electromagnetic energy (photons). Protons entering a gravitational field will radiate. The physics for protons is the same with the difference being charge and mass. Incidentally this is also why the electrons circularly orbiting the protons to create atoms - fails as a model.--DHeyward (talk) 13:09, 5 January 2013 (UTC)[reply]

Validating the Big Bang or is it unfalsifiable?

Correct me if I'm wrong, but the size of the observable universe appears to be at the limit of current telescopes is it not? But as the scopes get better, we should stop seeing more distant galaxies at some point, thus if this observable myopic cliff has not been definitively found as of yet, then I wouldn't think the Big Bang model has been validated. My question is this, if the newer scopes show us a bigger universe, will the Big Bang parameters simply get updated again and again such that its unfalsifiable or is there a breaking point? --Trillianthcircuit (talk) 06:50, 4 January 2013 (UTC)[reply]

There is no distance limit to a telescope. The reason we can't see much further is because there isn't all that much more to see. With better telescopes we'll see more detail and better what we see already. If there were galaxies extending out further away we'd have seen some. Parameters haven't been updated, if anything the estimate for the age of the universe has been reduced a bit since the original estimates from Hubble but not recently. What is seen at the far limits is different from what is around now and the big bang theory is quite well established. If more was seen it would mean a major change in science not just tweaking some parameters. Much as scientists would like a major upset like that along with the new money it would bring it does not seem at all likely. Dmcq (talk) 12:39, 4 January 2013 (UTC)[reply]
Well, there may not be a distance limit to a telescope, per se, but there's certainly a distance limit to light. If the light coming from faraway galaxies is too faint then there's a chance any instrument we come up with won't be able to detect it. No matter how good telescopes get, no one will ever be able to tell me how many finger's I'm holding up from an observation point in the Andromeda galaxy. Evanh2008 (talk|contribs) 12:52, 4 January 2013 (UTC)[reply]
The following may be of interest : http://skeptics.stackexchange.com/questions/2335/what-evidence-of-the-big-bang-theory-does-existSfan00 IMG (talk) 12:33, 4 January 2013 (UTC)[reply]
We can "see" (i.e. detect and map in considerable detail) the cosmic microwave background radiation, which consists (roughly - some hand waving here) of photons emitted at the end of recombination in the early universe. Beyond/before this point the universe was a hot dense plasma which was opaque to electromagnetic radiation. So in a sense we can already "see" as far as it will ever be possible to "see". Gandalf61 (talk) 12:58, 4 January 2013 (UTC)[reply]
The current estimate is based on the distance to galaxies in the deep field of Hubble in accordance with the Big Bang and the red shift. At one time, we could not see that far and did not know about expansion, but with larger and better scopes we have been able to see further. Thus to make theory fit the data, they added cosmic inflation and then cosmic acceleration. Hence, the model fits this data for now, and Hubble can't resolve fainter galaxies that might emerge in abundance with a larger telescope. Thus my question is not trivial because if we see even more faint galaxies, then based on previous model changes, I would expect that astronomers would simply increase their inflation parameter because the CMB data does not appear to stop them from indefinitely increasing this parameter. Another thing that bothers me, is the mantra that the inflation rate is somehow meaningless, and in a terribly ironic way they are right, for they would repeatedly increase the inflation rate indefinitely to accommodate more galaxies, thus supposing Eric Lerner is correct that the Big Bang is wrong, I am asking is there any other part of the model such that a repetitious parameter change could eventually break it, or could it be just an unending shell game? In addition, Dmcq and the articles mention that the nearby oldest galaxies differ from the more distant younger galaxies. That would certainly mean that our neighboring galaxy formations have a similar history, but this important information would be more informative if a summary of this data is given or displayed in an article and I would appreciate accessible references too. --Trillianthcircuit (talk) 14:56, 4 January 2013 (UTC)[reply]
Eric Lerner? Well, his version of Plasma cosmology has an alternative explanation of microwave background radiation, but other astronomers have found lot of problems with his theory (at the end of Eric_Lerner#The_Big_Bang_Never_Happened). Even if the Big Bang theory was proven to be wrong, Lerner's theory wouldn't become more accepted until those problems were solved. I think that Lerner hasn't made any successful prediction using his theory, which is another point against it. --Enric Naval (talk) 18:41, 4 January 2013 (UTC)[reply]
The OP's question is interesting; let me put it more succinctly:
Is the Big Bang Theory falsifiable? Specifically, what could we hypothetically observe that would be inconsistent with the Big Bang having occurred?
I've seen lots of discussion in various places about what observations that we have made are consistent with the theory; but how would we falsify it? I sort of remember that back in the 1990s there was substantial consternation about the fact that cosmologists dated the universe's beginning at around 13 billion years ago, but that the oldest stars appeared to be more like 19 billion years old. I recall that the paradox was resolved when people found better ways of estimating the ages of stars. Would it contradict the Big Bang theory if, hypothetically, we found some stars that are solidly dated at well over 14 billion years old? Duoduoduo (talk) 17:33, 4 January 2013 (UTC)[reply]
This would contradict the currently accepted standard cosmological model, and will falsify it. But it will not be sufficient to falsify the Big Bang because different models are possible. Ruslik_Zero 18:33, 4 January 2013 (UTC)[reply]
Of course big bang cosmology could have failed many tests, and not long ago it was thought to be on shaky ground, as you said. It dominates the field now because it continues to be consistent with large bodies of new experimental data that have ruled out every proposed alternative. In other words, it has succeeded in the same way as any other scientific theory. I encourage you to read Ned Wright's excellent cosmology site, in particular the FAQ which lists various independent sources of evidence for the big bang, evidence against some alternatives including Eric Lerner's model, and the tutorial which is the best explanation of big bang cosmology I've seen. -- BenRG (talk) 18:54, 4 January 2013 (UTC)[reply]
OK, so basically, if we do not reach a deep field resolution where we stop seeing more galaxies, then because the Big Bang appears correct for various reasons, this is not a problem because theorists would have no problem with increasing the expansion factor. Of course, that would be frustrating to not find the edge of the sea (unless you are an ancient mariner sailing one), but hardly newsworthy. --Modocc (talk) 20:29, 4 January 2013 (UTC)[reply]
The most distant thing we can see is the cosmic microwave background; it is older, hence more distant, than any galaxy we might see (according to big bang cosmology). Its uniformity is evidence that the universe is homogeneous out to that distance. So we already know that we can't see the edge of the "sea", in that sense. Since more distant visible objects are older, and galaxies didn't form until some time after the big bang, there should be a gap between the most distant visible galaxies and the distance of the CMBR. If we could determine that the galaxies ended closer or farther than that it would at least contradict current theories of structure formation.
The Hubble Deep Field images, if that's what you and the OP are talking about, are not a major piece of evidence for the big bang. Currently the most important evidence is the CMBR spectrum: it is far too uniform to be starlight, it has small inhomogeneities of the right size to explain later structure formation, and its angular power spectrum matches the rather complicated prediction of inflationary ΛCDM over a wide range of scales. It was the COBE data that first gave astronomers a high level of confidence in big bang cosmology. -- BenRG (talk) 22:05, 4 January 2013 (UTC)[reply]
The Hubble constant gives a rate of accelerating expansion of about 74 k/s/Mp, and back-of-the-envelope calculations suggest that that could easily allow for a recessional velocity of two objects relative to each other that is well in excess of the speed of light. This could hypothetically allow for there to be a cut-off point beyond which we could not see (since the objects beyond that point would be receding from us too quickly for their light to ever reach us), but the CMB observations pretty much rule that out entirely. Evanh2008 (talk|contribs) 01:34, 5 January 2013 (UTC)[reply]

Frame of reference

  1. Is it possible to change a non-inertial frame to an inertial frame or an inertial frame to a non-inertial frame ?
  2. Is it possible for the same frame to appear as inertial as well as non-inertial for two different observers or for only one observer ?
  3. In which year it was claimed that carbon form more number of compounds than any other element ?
  4. In which standard (class) calculus, space-time, and theory of relativity is taught to the students of USA ? Want to be Einstein (talk) 12:11, 4 January 2013 (UTC)[reply]
For #4, calculus is often taught in several classes over a year or two (depending on the depth), starting often in a student's last year of high school or their first year of college. Space-time and relativity are often covered in very brief overview in introductory (high school or first year of college) physics classes, but generally aren't covered in detail until later years of college. --Jayron32 13:04, 4 January 2013 (UTC)[reply]

I asked question #4 because I am a student of standard 9 and I have read much more about these topics from books. Nowadays I am reading on these topics from Wikipedia. There is enormous information about these topics on Wikipedia. Want to be Einstein (talk) 13:18, 4 January 2013 (UTC)[reply]

The difference between a non-inertial frame and an inertial one is that the former is accelerating where the latter is not. You can't "change" one into another just by changing your point of view or using some mathematical trickery. They really are different. If you're in a rocketship and you feel yourself being pushed back into your seat (or whatever) then you're in a non-inertial frame. When you cut the rocket motors and coast, then you're no longer accelerating so you're in an inertial frame of reference - and you can immediately tell that this is the case by the fact that you're now floating free of your seat. Because such a simple experiment allows you to know which kind of frame of reference you're in - there is no way to "change" one into the other...except by turning off your engine. SteveBaker (talk) 17:38, 4 January 2013 (UTC)[reply]

bitcoin

I read the bitcoin article and think I have understood proof of work concept. A bit coin is a really long number which produces a hash which has quite a number of zeros. Ok, so bitcoins are really hard to find. But once I find the really long number, what is there to prevent me from transferring it to two people? How is a bit coin uncopiable? How is the transfer of ownerhsip ensured? Does it depend on the asumption of good faith of the bit coin clients? Cplusplusboy (talk) 15:14, 4 January 2013 (UTC)[reply]

The bitcoin approach requires a public consensus about a database; all transactions are recorded publically in a block cipher chain. If you make a transaction, and fail to announce it to the network, the transaction doesn't validate against the history of all prior bitcoin transactions; so phony transactions are "tamper-evident." This still requires some non-trivial technical effort: each transaction needs to be published and validated, and a community consensus is required to arbitrate which database stores the canonical record of transactions when there are conflicts. Nimur (talk) 16:25, 4 January 2013 (UTC)[reply]
And a bitcoin itself is not a hash or anything else. It is simply the result of transactions between bitcoin wallets. Transactions are from one wallet to another, and the miners do the work of putting transactions into the block chain and verifying things. This is computationally intensive, so the miners are rewarded with transactions that create new bitcoins, or eventually with transaction fees. 209.131.76.183 (talk) 17:51, 4 January 2013 (UTC)[reply]
Ah, it seems I had not understood the block cipher chain thing and 10 minute delay in txactions.. I'm currently going through their wiki, which seems to have more details [[21]]. — Preceding unsigned comment added by Cplusplusboy (talkcontribs) 09:08, 5 January 2013 (UTC)[reply]

Coastal armoring thoughts

What is causing more coastal erosion, climate change induced sea-level rise or coastal armoring? Does anyone have any thoughts on this question? Thanks. This is an honors thesis research question-not a homework question. — Preceding unsigned comment added by 99.146.124.35 (talk) 18:39, 4 January 2013 (UTC)[reply]

"Coastal armoring induced sea-level rise" ? Please explain. Are you talking about the sea level rising due to dropping concrete blocks and such into the ocean along the coast ? If so, that rise would be absurdly small, far less than a millimeter. Now, coastal armoring can cause erosion, but not by that mechanism. Basically it can redirect the force of waves onto a different area, and cause that area to erode more quickly. StuRat (talk) 20:02, 4 January 2013 (UTC)[reply]

I meant to say "climate change induced sea level rise or coastal armoring" -anon

I see you changed your original post. Coastal armoring presumably helps more than it hurts, or they wouldn't do it. So, in that respect it does less harm, although, in one particular spot, coastal armoring may cause a worse problem than global climate change. StuRat (talk) 05:47, 5 January 2013 (UTC)[reply]

what is the pH of borate-carbonate-bicarbonate solution? why is no formula given for the pH calculation of a hetereogeneous buffer solution? is the pH of an amino acid solution its pI?

What is the general solution for a heterogeneous buffer like ammonium acetate? Somehow, the folks who wrote buffer solution seem to think that all the interesting buffers worthy of discussion only consist of weak acids and their conjugate bases. Buffer solution doesn't discuss heterogeneous solutions at all. What is the general solution for buffering pH with multiple polyprotic substances?

The reason is that my friend and I are trying to reverse-engineer a proprietary aquarium reef buffer sold by Seachem which has a buffering pH of 8.3. It cannot contain any ammonium/ammonia or phosphates (this would be detrimental to the aquarium ecology). I was thinking it was a mixture of sodium borate and bicarbonate/carbonate, but I don't know how to determine whether the buffering pH would be 8.3.

Also, how do I calculate the pI of a solution containing multiple amino acids (how does ProtParam do it?) as well as the buffering pH? 72.229.155.79 (talk) 18:58, 4 January 2013 (UTC)[reply]

The calculation for a heterogeneous buffer solution will simply be the simultaneous solution of the equilibrium equations for the two systems. If you're using the technique in the Buffer solution article:
ICE table for a two monoprotic buffers
[HA] [A-] [H+] [HB] [B-]
I C0 0 y D0 0
C -x x x+z -z z
E C0-x x y+x+z D0-z z
Working the equilibrium constant equations as before will give you two equations, one for Ka-A and one for Ka-B. You can then solve one for the change variable (e.g. solve the Ka-B equation for z), and then substitute into the other, and then you can solve for the other change variable. Extend to however many species you have. Analytical techniques may be a bit tricky, so you may need to resort to numerical calculations. Similar approaches can be used for multiple polyprotic buffers.
At some point, though, it gets easier to work forward from the Henderson–Hasselbalch equations. Assume a change in pH from A to A±x, and then calculate what the change in species concentration (and thus the added H+) would have to have been to make the change. If all you're concerned about is how well a solution will hold a pH of 8.3, that's probably the better approach, anyway.
What you'll find is that unless the desired pH is within about one pH unit of one of thee pKa's of one of the buffers (ignoring the Bicarbonate buffering system gas-exchange issue), you'll not get appreciable buffering - if A doesn't buffer by itself, and B doesn't buffer by itself, the A+B combination won't do anything special unless they interact somehow. (And amino acids aren't special - treat them just like any other polyprotic acid) -- 205.175.124.30 (talk) 04:36, 5 January 2013 (UTC)[reply]

Syrian missiles

Do Syrian war planes have laser guided missiles/bombs? If not, how accurate are unguided missiles/bombs? Is it any better than the unguided missiles/bombs of WW2--Jonharley667 (talk) 19:51, 4 January 2013 (UTC)[reply]

This site Xairforces.net lists; "• Air-to-ground-missiles (ASM):
AS-7 Kerry (Kh-23 general purpose), AS-9 Kyle ( Kh-28 confusional), AS-10 Karen (H-25ML/MR model - a general purpose / Kh-25MP antiradar), AS-11 Kilter (Kh-58 antiradar), AS-12 Kegler (Kh-25PS confusional), AS-14 Kedge (Kh-29 general purpose), AT-10 Stabber, (AT-2B Swatter B (9M17M Skorpion-M MCLOS 9K8 (Falanga-M)); ATGM helicopter Mi-24/25), HOT (ATGM for Gazelle helicopters)" .
Of those the AS-10 Karen, AS-12 Kegler AS-14 Kedge are laser guided. So they do have them, but if you're trying to get a few people hiding in a ruined house, I would think that you would go for a cheaper option, like unguided rockets or free-fall bombs. Are they more accurate than the WWII versions? I suspect not very much, but I couldn't find any hard data. Alansplodge (talk) 21:09, 4 January 2013 (UTC)[reply]
(I'm not sure how this wasn't marked as an edit conflict, but the answer above is probably going to be more helpful.) The Syrian Air Force article has a list of their aircraft; look for the "Combat aircraft" and "Attack helicopter" sections for attack aircraft. Of those craft, the Mikoyan MiG-29, the Sukhoi Su-24, and the Sukhoi Su-17 are compatible with laser-guided weapons, according to their articles. From my quick search of Wikipedia on the subject, I'm only finding specific references to more advanced air forces, such as those of the US, Israel, and India, using laser-guided missiles, but I'm not too familiar with the subject myself. This could reflect the encyclopedia's systemic bias. --BDD (talk) 21:11, 4 January 2013 (UTC)[reply]

Are they able to acquire laser guided bombs though, even if the plane accepts them? In Libya their planes could accept them but it didn't seem they used them as war footage of Libyan jet runs showed high inaccuracy. Also is a laser guided missile the same thing as a laser guided bomb? --Jonharley667 (talk) 01:21, 5 January 2013 (UTC)[reply]

See our article Laser-guided bomb which explains how it works. I'm not sure if the Syrians have them, or use them, but it looks as though some of their aicraft are capable of using them. But as I said before, cost is going to be a critical factor for a poor country in a long war. Laser Guided Bombs (LGB) are cheaper than Air to Surface Missiles (ASM) but unguided bombs are cheaper than either. Alansplodge (talk) 02:00, 5 January 2013 (UTC)[reply]
Unguided bombs are not cheap for use against specific targets, only against large areas. You need to use a lot of them to have any chance of hitting a target. Dmcq (talk) 04:46, 5 January 2013 (UTC)[reply]
Modern aircraft with Continuously Computed Impact Point bombsights (such as the F-15 Eagle) can deliver unguided bombs with almost as much precision as guided bombs (using a variation of the dive bombing tactic), but this requires a highly skilled pilot -- which is why you never see the Syrians achieve this kind of precision. 24.23.196.85 (talk) 05:05, 5 January 2013 (UTC)[reply]
Constantly computed impact point seems to get there. I'm surprised they don't have the system autopilot the plane, perhaps they do now for all I know. That would reduce the skill requirements. I'd have thought they would still need to get fairly close to have a good chance of hitting a building but I'm no expert. Dmcq (talk) 14:18, 5 January 2013 (UTC)[reply]
It is not unusual for export version of warcraft to have "different" capabilities than those used by the designing nations. Rmhermen (talk) 15:35, 5 January 2013 (UTC)[reply]
(ec-wtf?)
The superpowers with sophisticated systems like CCIP have highly trained personnel. Autopilot would be redundant to them, and it would open a window of vulnerability if the plane was shot at with the autopilot on. Basically, a pilot employing CCIP would try to go into a zero-g dive for a moment, to make the impact point stabilize on the target, and then drop an unguided bomb, or in a high-threat environment, not dive at all and drop a volley ("ripple") of bombs "across" the target, to nail it at least once in the process.
And yes, exported versions are usually limited when it comes to weapon compatibility. - ¡Ouch! (hurt me / more pain) 18:18, 5 January 2013 (UTC)[reply]


So what kind of laser guided bombs do the Syrians have? And are their jets equipped with Constantly computed impact point?--Jonharley667 (talk) 19:00, 5 January 2013 (UTC)[reply]

Railroad switch question

The railroad switch article contains this:

Joints are used where the moving points meet the fixed rails of the switch. They allow the points to hinge easily between their positions. Originally the movable switch blades were connected to the fixed closure rails with loose joints, but since steel rails are somewhat flexible it is possible to make this join by thinning a short section of the rail itself.

Points/switches on model rail layouts have, I believe, pinned hinges at the place in question, and the wording above implies that this used to be the case in full-size operation but that now the steel rail used will flex each time the points are changed. Am I right in assuming that any failure due to metal fatigue would take far longer to occur than the normal lifetime of the rail, brought to an end by the wear caused by train movement over it? — Preceding unsigned comment added by 31.54.246.112 (talk) 20:47, 4 January 2013 (UTC)[reply]

As long as the stress level stays well below the yield point, the fatigue life of a typical metal part is measured in millions of cycles. For steel parts, if the stress level is below the fatigue limit, the fatigue life is infinite.
(If the stress level is above the yield point, the fatigue life can be as low as tens of cycles, as demonstrated in the Grayrigg derailment, where the spreader bars for a switch went from "passes visual inspection" to "complete failure" in less than twelve days.) --Carnildo (talk) 02:50, 5 January 2013 (UTC)[reply]
That could just mean the visual inspection was insufficient to detect the micro-cracks. StuRat (talk) 05:43, 5 January 2013 (UTC)[reply]

January 5

"Acute in age"

I recently woke up with pain in my finger. When I went to the doctor and had an x-ray, I learned that I had somehow broken it (probably overnight on the 31st). The doctor printed up a little document explaining what happened, but I didn't look at it until I had already left (because he explained very well verbally to me). On the document, the text "fracture radiographically appears acute in age" appears. What does the phrase "acute in age" mean? Google is of no help. Thanks! -- Tohler (talk) 00:03, 5 January 2013 (UTC)[reply]

Well, "acute injury" means that it stems from a single traumatic event [22], rather than chronic (stemming from some long-term cause). -- Finlay McWalterTalk 00:07, 5 January 2013 (UTC)[reply]
See also the paragraph of Acute (medicine) beginning "The acute phase of an injury is the period of time in between when the injury is sustained, and the beginning of the sub-acute phase." -- Finlay McWalterTalk 00:11, 5 January 2013 (UTC)[reply]
In other words, "acute in age" = "recent injury". 24.23.196.85 (talk) 04:34, 5 January 2013 (UTC)[reply]
Why be direct when they can be obscure? Like the lawyer who frets over your not writing a will, warning you that you could die intestate. ←Baseball Bugs What's up, Doc? carrots05:56, 5 January 2013 (UTC)[reply]
Often laypeople get the urge to denigrate technical terminology, saying it could have been replaced with some lay terminology that doesn't actually mean the same precise thing. Maybe some of the above comments are an example of that. It looks to me like "acute in age" means "still in the acute stage, not yet in the sub-acute stage", in which case "age" refers to how far along it is in the expected sequence, not how recent in clock or calendar time it is (even though those would be imperfectly correlated). Note that the doctor was not being obscure, since he explained it well orally to the OP. Duoduoduo (talk) 14:07, 5 January 2013 (UTC)[reply]
"Acute" as a word means "sharp". In fact, it appears that the term "acute" is used in at least two somewhat different ways in the medical community. Kind of like how football uses the term "safety". ←Baseball Bugs What's up, Doc? carrots16:00, 5 January 2013 (UTC)[reply]
Three actually, given there's "player safety" (prevention of bodily injury) as well as the positions and the scoring method. --Jayron32 19:13, 5 January 2013 (UTC)[reply]

Science Olympiad

my teacher is not able to help me with these because we don't have any astronomy class in my school. I got stuck on number 27, 14, 15. see here. anskey, I got the answers but that doesn't mean I know how to do them. I need help on show me how to do it like what formula do I need to use, what can I tell from the result after calculating the formula, how do I interpret that? This is not homework as obvious.174.20.15.246 (talk) 01:27, 5 January 2013 (UTC)[reply]

Number 27 tells you the period (25 days) and apparent magnitude (mean = +24.9) of a Cepheid variable. Use the equation at Classical Cepheid variable to calculate the absolute magnitude from the period, then the equation here to calculate the distance. --Jayron32 01:38, 5 January 2013 (UTC)[reply]
Alright got it! How about 14, 15?174.20.15.246 (talk) 02:05, 5 January 2013 (UTC)[reply]
14 and 15 can be solved with the Tully-Fisher relation. --99.227.0.168 (talk) 07:25, 5 January 2013 (UTC)[reply]

Lever frame

On British railroads, the levers in a lever frame are (well, used to be) color-coded as follows: Red -- Home signal; Yellow -- Distant signal; Black -- Railroad switch; Blue -- Facing point lock (locks corresponding switch in place); Brown -- Level crossing gate; White -- Spare. Does anyone know what were the corresponding color-coding conventions in France and Germany in the 1940s? Thanks in advance! 24.23.196.85 (talk) 05:26, 5 January 2013 (UTC)[reply]

Recession

Do you think the world recession is affecting science and engineering? — Preceding unsigned comment added by 90.219.64.4 (talk) 14:35, 5 January 2013 (UTC)[reply]

Maybe in cases where research and development budgets are being cut. In my company, R&D is actually expanding. Companies figure R&D is vital for future competitive advantage. ←Baseball Bugs What's up, Doc? carrots15:54, 5 January 2013 (UTC)[reply]

sleeping with makeup on -- does your skin really need the oxygen?

I am transgender, but I also study biochemistry, so I'm always skeptical of the skincare industry. I like to leave makeup on because I can still wake up looking female (especially if I am sleeping with someone) but I wonder (despite what I read on the internet) whether this is really all that bad. Your epidermis isn't really alive, right, and all the alive parts get enough oxygen from their blood?

Not a request for medical advice, more of an inquiry of whether there are skin cells (that are alive) that need oxygen from the exterior. 72.229.155.79 (talk) 18:43, 5 January 2013 (UTC)[reply]

No, they don't need oxygen, or you couldn't wear a bandage for very long. The problem is that some chemicals will irritate or block pores if left on too long. You can see if the product has a tollfree number to call for advice. μηδείς (talk) 18:53, 5 January 2013 (UTC)[reply]
What are the consequences of blocked pores? To me it seems it would just stop sweat. 72.229.155.79 (talk) 19:19, 5 January 2013 (UTC)[reply]